87
Anota¸c˜ oes sobre s´ eries Rodrigo Carlos Silva de Lima rodrigo.uff[email protected] 27 de julho de 2014

Series

Embed Size (px)

Citation preview

Page 1: Series

Anotacoes sobre series

Rodrigo Carlos Silva de Lima ‡

[email protected]

27 de julho de 2014

Page 2: Series

1

Page 3: Series

Sumario

1 Series 4

1.1 Notacoes . . . . . . . . . . . . . . . . . . . . . . . . . . . . . . . . . . . . . 4

1.2 Definicao e conceitos basicos . . . . . . . . . . . . . . . . . . . . . . . . . . 4

1.2.1 Mudanca de variavel em series . . . . . . . . . . . . . . . . . . . . . 6

1.2.2 Condicao necessaria para convergencia de series . . . . . . . . . . . 8

1.2.3 Criterio de comparacao . . . . . . . . . . . . . . . . . . . . . . . . . 12

1.2.4 Criterio de condensacao de Cauchy . . . . . . . . . . . . . . . . . . 14

1.2.5 Series do tipo∞∑k=1

1

kpe divergencia da serie harmonica. . . . . . . . 15

1.2.6 Divergencia da serie harmonica. . . . . . . . . . . . . . . . . . . . . 18

1.2.7 Divergencia de∞∑k=1

1

kpcom p < 1. . . . . . . . . . . . . . . . . . . . 19

1.2.8 Series de funcoes racionais . . . . . . . . . . . . . . . . . . . . . . . 23

1.2.9 Criterio de Cauchy para series . . . . . . . . . . . . . . . . . . . . . 25

1.3 Series absolutamente convergentes . . . . . . . . . . . . . . . . . . . . . . . 27

1.3.1 Toda serie absolutamente convergente e convergente . . . . . . . . . 27

1.3.2 Parte negativa e positiva de uma serie . . . . . . . . . . . . . . . . 28

1.3.3 Teste da raiz-Cauchy . . . . . . . . . . . . . . . . . . . . . . . . . . 31

1.3.4 Teste da razao-D’ Alembert . . . . . . . . . . . . . . . . . . . . . . 33

1.3.5 Criterio de Dirichlet . . . . . . . . . . . . . . . . . . . . . . . . . . 38

1.3.6 Criterio de Leibniz . . . . . . . . . . . . . . . . . . . . . . . . . . . 41

1.3.7 Criterio de Kummer . . . . . . . . . . . . . . . . . . . . . . . . . . 46

1.4 Comutatividade . . . . . . . . . . . . . . . . . . . . . . . . . . . . . . . . . 48

1.5 Soma sobre um conjunto infinito arbitrario . . . . . . . . . . . . . . . . . . 52

1.6 Series em espacos vetoriais normados . . . . . . . . . . . . . . . . . . . . . 53

1.7 Soma de Cesaro . . . . . . . . . . . . . . . . . . . . . . . . . . . . . . . . . 53

2

Page 4: Series

SUMARIO 3

1.7.1 Serie de Grandi . . . . . . . . . . . . . . . . . . . . . . . . . . . . . 59

1.8 Sequencias (C,P ) somaveis . . . . . . . . . . . . . . . . . . . . . . . . . . 60

1.9 Series de termos nao-negativos . . . . . . . . . . . . . . . . . . . . . . . . . 60

1.9.1 Criterio de comparacao por limite para series de termos positivos . 61

1.10 Representacao decimal . . . . . . . . . . . . . . . . . . . . . . . . . . . . . 67

1.11 Teste da integral para convergencia de series . . . . . . . . . . . . . . . . . 74

1.11.1 Sequencia de variacao limitada . . . . . . . . . . . . . . . . . . . . 76

1.12 Series em espacos vetoriais normados . . . . . . . . . . . . . . . . . . . . . 79

1.13 Produto de series . . . . . . . . . . . . . . . . . . . . . . . . . . . . . . . . 81

1.14 Series e desigualdade das medias . . . . . . . . . . . . . . . . . . . . . . . . 84

1.15 Extensao do conceito de serie para−∞∑k=1

ak. . . . . . . . . . . . . . . . . . . 84

Page 5: Series

Capıtulo 1

Series

Esse texto ainda nao se encontra na sua versao final, sendo, por enquanto, cons-

tituıdo apenas de anotacoes informais. Sugestoes para melhoria do texto, correcoes da

parte matematica ou gramatical eu agradeceria que fossem enviadas para meu Email

[email protected].

1.1 Notacoes

Usaremos o ∆ para simbolizar o operador que faz a diferenca de termos consecutivos

de uma funcao

∆f(x) := f(x+ 1)− f(x).

A notacao Q para denotar o operador que faz o quociente,

Qf(x) =f(x+ 1)

f(x).

1.2 Definicao e conceitos basicos

Vamos definir o somatorio como

s∑k=s

f(k) = f(s) ∀ s ∈ Z

b∑k=a

f(k) =

p∑k=a

f(k) +b∑

k=p+1

f(k) ∀ b, a, p ∈ Z.

4

Page 6: Series

CAPITULO 1. SERIES 5

Perceba que nao colocamos limitacao em b, a e p inteiros , na definicao acima podemos

ter b < a. Em especial tomando p = a− 1 na identidade acima segue que

b∑k=a

f(k) =a−1∑k=a

f(k) +b∑

k=a

f(k)

logo deve valera−1∑k=a

f(k) = 0 que e chamada de soma vazia .

m Definicao 1 (Serie). Sejam a ∈ Z, A um conjunto indutivo que contenha a ,

f(k) : A→ R uma funcao . Chamamos de serie o limite do somatorio

lim s(n) = limn∑

k=a

f(k) :=∞∑k=a

f(k)

, caso o limite exista, onde

s(n) =n∑

k=a

f(k).

Se existir o limite de s(n) com lim s(n) = s diremos que a serie e convergente e sua soma

e s.

Se o limite lim s(n) nao existir diremos que a serie diverge. A soma finita s(n) =n∑

k=a

f(k) e chamada reduzida de ordem n ou n−esima soma parcial da serie∞∑k=a

f(k) . Se

a serie e divergente, pode acontecer de lim s(n) = ∞, lim s(n) = −∞ ou a soma oscilar1.

Se (sn) converge diremos que∞∑k=a

f(k) converge caso (sn) seja divergente diremos que

∞∑k=a

f(k) diverge , apesar de∞∑k=a

f(k) ser um numero real, caso haja convergencia e nao

haver numero associado a∞∑k=a

f(k) caso haja divergencia, tal uso e feito apenas no sentido

de notacao . Caso a serie seja convergente dizemos tambem que (f(k)) e somavel .

b Propriedade 1. Toda sequencia (xn) de numeros reais pode ser considerada como a

sequencia das reduzidas de uma serie.

ê Demonstracao. Supondo

xn =n∑

k=1

ak

1Quando (s(n)) diverge e lims(n) = ∞ e lims(n) = −∞.

Page 7: Series

CAPITULO 1. SERIES 6

aplicando ∆ segue

∆xn = an+1

e para n = 1, x1 =1∑

k=1

ak = a1, se n = 0 temos x0 =0∑

k=1

ak = 0 por ser uma soma vazia

n∑k=1

ak =n−1∑k=0

ak+1 =n−1∑k=0

∆xk = xk

∣∣∣∣n1

= xn − x0 = xn.

Se ∆xn = an+1 nao implica que an = ∆xn−1, pois a primeira vale para n ≥ 0 natural

a segunda nao vale para n = 0.

Z Exemplo 1. Encontrar o erro na manipulacao

0 = 0 + 0 · · · =

= (1− 1) + (1− 1) + · · · =

= 1 + (−1 + 1) + (−1 + 1) + · · · = 1

logo 1 = 0.

Comecamos com uma serie∞∑k=1

ak onde cada ak = 0 = 1 − 1, isto e, a soma dos

elementos da sequencia (0, 0, · · · ) entao ate a segunda linha tudo esta correto, porem na

terceira linha tratamos o termo da serie somada como os termos da sequencia (1, −1 +

1, −1 + 1, · · · ) que e uma serie diferente da serie inicial

1.2.1 Mudanca de variavel em series

b Propriedade 2 (Mudanca de variavel em series). Por mudanca de variavel temos que

se g(n) =n∑

k=a

f(k) entao g(n) =n+t∑

k=a+t

f(k−t) com limn = ∞ temos tambem limn+t = ∞

logo

limn∑

k=a

f(k) = limn+t∑

k=a+t

f(k − t) =∞∑k=a

f(k) =∞∑

k=a+t

f(k − t).

Logo se temos uma serie∞∑k=a

f(k) podemos somar t aos limites (t +∞ = ∞, t + a),

subtraindo t do argumento da funcao

∞∑k=a

f(k) =∞∑

k=a+t

f(k − t).

Page 8: Series

CAPITULO 1. SERIES 7

b Propriedade 3 (Produto por −1). Por propriedade de somatorios se g(n) =n∑

k=a

f(k)

entao g(n) =−a∑

k=−n

f(−k) com limn = ∞ temos lim−n = −∞ e

limn∑

k=a

f(k) = lim−a∑

k=−n

f(−k) =∞∑k=a

f(k) =−a∑

k=−∞

f(−k).

∞∑k=a

f(k) =−a∑

k=−∞

f(−k).

b Propriedade 4. Sejam∞∑k=a

f(k) e c um numero real diferente de zero entao∞∑k=a

f(k)

e convergente sse∞∑k=a

cf(k) e convergente.

ê Demonstracao. Se g(n) =n∑

k=a

f(k) e convergente, existe o limite lim g(n) , vale

tambem c.g(n) = cn∑

k=a

f(k) =n∑

k=a

c.f(k) e existe o limite lim c.g(n) = c lim g(n) impli-

cando que a serie∞∑k=a

cf(k) = c

∞∑k=a

f(k) e convergente.

Se h(n) =n∑

k=a

cf(k) = cg(n) entao g(n) =n∑

k=a

f(k), sendo h(n) convergente, entao

limh(n) = d para algum d real e vale limh(n)

c= lim g(n) como c = 0 tem-se lim g(n) =

limh(n)

c=d

cque existe de onde segue que lim g(n) =

∞∑k=a

f(k) e convergente.

b Propriedade 5. Sejam∞∑

k=as

fs(k) convergente pra toda expressao fs(k), gs(n) =

n∑k=as

fs(k) , as numeros inteiros e cs numeros reais, para todo s ∈ [1, p]N , entao

p∑s=1

cs

∞∑k=as

fs(k)

converge.

ê Demonstracao. Considerando a soma

p∑s=1

csgs(n) como os limites lim gs(n) exis-

tem e pela propriedade de soma de limites segue que existe o limite:

lim

p∑s=1

csgs(n) =

p∑s=1

cs lim gs(n) =

p∑s=1

∞∑k=as

fs(k).

Page 9: Series

CAPITULO 1. SERIES 8

1.2.2 Condicao necessaria para convergencia de series

b Propriedade 6 (Condicao necessaria para convergencia de series). Se s(n) =n∑

k=a

ak

converge entao lim ak = 0.

ê Demonstracao. Temos que se lim a(n) = s e tambem lim s(n+ 1) = s e

s(n+ 1)− s(n) =n+1∑k=a

ak −n∑

k=a

ak = an+1

logo

lim s(n+ 1)− s(n) = lim an+1 = lim s(n+ 1)− lim s(n) = s− s

assim lim an+1 = 0, lim an = 0.

Essa e uma condicao necessaria porem nao suficiente para convergencia de series.

$ Corolario 1. Se f(k) nao tende a zero a serie nao pode convergir. Esse criterio e

util para provar que algumas series divergem. Veremos depois que esse criterio nao e

suficiente, pois existem series em que o termo somado tende a zero mas a serie diverge,

como e o caso da serie harmonica.

b Propriedade 7. Se∞∑k=1

ak e convergente entao∞∑k=1

a2k + a2k−1 e convergente e tem

mesma soma que a primeira serie.

ê Demonstracao.

Seja sn =n∑

k=1

ak, ela converge, entao s2n =2n∑k=1

ak =n∑

k=1

a2k +n∑

k=1

a2k−1 =n∑

k=1

a2k +

a2k−1 tambem converge e tende ao mesmo limite de sn.

Z Exemplo 2. A serie∞∑k=1

a2k + a2k−1 pode convergir porem∞∑k=1

ak, como e o caso de

tomar ak = (−1)k a serie∞∑k=1

(−1)k nao converge pois lim(−1)k = 0, porem a2k + a2k−1 =

1− 1 = 0 e a primeira serie converge.

b Propriedade 8. A serie∞∑k=a

f(k) converge ⇔ a serie∞∑k=b

f(k) converge. Esta propri-

edade nos diz que o estado de convergencia da serie nao e alterado pela reducao ou adicao

de um numero finito de termos, isto e, podemos alterar o limite inferior do somatorio por

outro numero real e a convergencia da serie nao se altera.

Page 10: Series

CAPITULO 1. SERIES 9

ê Demonstracao.Tomamos g(n) =n∑

k=a

f(k) e h(n) =n∑

k=b

f(k). Se b = a nao temos

nada a mostrar, pois as series serao iguais. Se b > a tem-se

g(n) =n∑

k=a

f(k) =b−1∑k=a

f(k) +n∑

k=b

f(k) =b−1∑k=a

f(k) + h(n)

g(n)−b−1∑k=a

f(k) = h(n)

supondo g(n) convergente e tomando o limite n → ∞ temos que no lado esquerdo te-

mos uma serie convergente e no lado direito a serie tambem sera convergente, se h(n) e

convergente, usamos que

g(n) =b−1∑k=a

f(k) + h(n)

tomando o limite tem-se que h(n) convergente implica g(n) convergente. Se a > b usamos

o mesmo procedimento

h(n) =n∑

k=b

f(k) =a−1∑k=b

f(k) +n∑

k=a

f(k) =a−1∑k=b

f(k) + g(n). (1.1)

h(n)−a−1∑k=b

f(k) = g(n) (1.2)

se g(n) converge usamos 1.1 se h(n) converge usamos 1.2.

Como o limite inferior do somatorio nao altera na convergencia, iremos em alguns

momentos denotar a serie sem o limite inferior, da seguinte maneira

∞∑k

f(k) =∞∑f(k)

Z Exemplo 3 (Serie geometrica). Vamos estudar a convergencia da serie

∞∑k=0

ak.

Se a = 1 temos a soman∑

k=0

1 = n+ 1, limn∑

k=0

1 = ∞.

Se a = 1 temosn−1∑k=0

ak =ak

a− 1

∣∣∣∣n0

=an − 1

a− 1

Page 11: Series

CAPITULO 1. SERIES 10

quando a > 1 o limite lim an = ∞, com a < −1 a sequencia alterna valores tomando

valores positivos para valores pares de n e negativos para valores ımpares de n, porem

com valor absoluto crescente, o limite nao existe nesse caso. Caso a = −1 o resultado da

soma finita en−1∑k=0

(−1)k =(−1)n − 1

−2

a sequencia alterna entre valor 0 para n par e 1 para n ımpar. Se |a| < 1 tem-se que

lim an = 0 e o resultado da serie e

∞∑k=0

ak = liman − 1

a− 1=

−1

a− 1=

1

1− a.

Podemos usar tambem a condicao necessaria para convergencia de series. Temos que

ter lim an = 0 , isto so acontece quando |a| < 1, entao estes sao os unicos valores de a

para os quais a serie e convergente.

Z Exemplo 4. A serie∞∑k=0

a2

(1 + a2)k

converge com qualquer a ∈ R. Vale que 1 ≤ a2+1 ∀ a ∈ R logo 0 <1

1 + a2≤ 1, portanto

a serie converge por ser serie geometrica. Sabemos que∞∑k=0

bk =1

1− b, substituindo

b =1

a2 + 1, chegamos no resultado

∞∑k=0

1

(1 + a2)k=a2 + 1

a2⇒

∞∑k=0

a2

(1 + a2)k= a2 + 1.

Z Exemplo 5. Mostrar que a serie

∞∑n=a

(−1)nann!

onde an =n∏

k=1

2k diverge. Vamos chegar primeiro numa expressao para o termo geral

an =n∏

k=1

2k =n∏

k=1

2n∏

k=1

k = 2n.n!

Page 12: Series

CAPITULO 1. SERIES 11

logo a serie e∞∑n=a

(−1)n2nn!

n!=

∞∑n=a

(−1)n2n

sendo bn = (−1)n2n o limite lim bn = lim(−1)n2n = 0 o limite nao existe pois a sub-

sequencia b2n = 22n tem limite +∞ e a subsequencia b2n+1 = −22n+1 tem limite −∞.

Z Exemplo 6. Dadas as series∞∑k=1

ak e∞∑k=1

bk com an =√n+ 1−

√n , bn = log(1+

1

n)

, mostre que lim an = lim bn = 0. Calcule explicitamente as n-esimas reduzidas sn e tn

destas series e mostre que lim sn = lim tn = +∞.

sn =n∑

k=1

ak =n∑

k=1

√k + 1−

√k =

n∑k=1

∆√k =

√k

∣∣∣∣n+1

1

=√n+ 1− 1

logo lim sn = ∞

tn =n∑

k=1

log(1+1

k) =

n∑k=1

log(k+1)−log(k) =n∑

k=1

∆log(k) = log(k)

∣∣∣∣n+1

1

= log(n+1)−log(1) = log(n+1)

logo lim tn = +∞. O limite dos termos das series

an =√n+ 1−

√n =

1√n+ 1 +

√n

lim an = 0

bn = log(1 +1

n)

0 < log(1 +1

n) =

log[(1 + 1n)n]

n≤

(1 + 1n)n

n

como lim(1+1

n)n = e entao tal sequencia e limitada, logo lim

(1 + 1n)n

n= 0 de onde segue

por teorema do sanduıche que lim log(1 +1

n) = 0. Usamos que log(n) < n. Assim temos

duas serie cujos termos gerais tendem a zero, porem as series divergem, esse exemplo

mostra que a condicao de lim f(k) = 0 em uma serie∞∑k=b

f(k) ser satisfeita nao garante

que a serie sera convergente, a condicao e apenas uma condicao necessaria.

b Propriedade 9. Seja (ak) sequencia com ak ≥ 0 ∀ k ou ak ≤ 0 ∀ k. Nessas condicoes

a serie∞∑k=a

ak converge ⇔ s(n) =n∑

k=a

ak forma uma sequencia limitada.

Page 13: Series

CAPITULO 1. SERIES 12

ê Demonstracao. ⇒). Seja s(n) limitada com ak ≥ 0 ∀ k , temos que

s(n+ 1)− s(n) = an+1 ≥ 0 ⇒ s(n+ 1) ≥ s(n)

assim s(n) e uma sequencia crescente limitada superiormente, portanto e convergente. Se

ak ≤ 0 temos

s(n+ 1)− s(n) = an+1 ≤ 0 ⇒ s(n+ 1) ≤ s(n)

logo s(n) sendo limitada inferiormente e decrescente e convergente.

⇐).

Agora se a serie e convergente entao s(n) e limitada , pois toda sequencia convergente

e limitada.

m Definicao 2. Quando temos ak ≥ 0 e s(n) =n∑

k=a

ak e limitada superiormente temos

que a serie∞∑k=a

ak converge, entao neste caso escrevemos∞∑k=a

ak <∞ para simbolizar que

a serie∞∑k=a

ak com ak ≥ 0 e convergente.

1.2.3 Criterio de comparacao

b Propriedade 10 (Criterio de comparacao). Sejam∞∑k=a

ak e∞∑k=a

bk series de termos

nao negativos. Se existem c > 0 e n0 ∈ N tais que ak ≤ cbk para todo k ≥ n0 entao :

1. A convergencia de∞∑k=a

bk implica a convergencia de∞∑k=a

ak .

2. A divergencia de∞∑k=a

ak implica a divergencia de∞∑k=a

bk.

ê Demonstracao.

1. De ak ≤ cbk seguen∑

k=n0

ak︸ ︷︷ ︸s(n)

≤ c

n∑k=n0

bk︸ ︷︷ ︸:=p(n)

sen∑

k=a

bk converge entaon∑

k=n0

bk converge de onde segue que s(n) e limitada supe-

riormente e como e crescente s(n) converge implicando a convergencia de∞∑k=a

bk

.

Page 14: Series

CAPITULO 1. SERIES 13

2. Agora se s(n) diverge, como e crescente seu limite e infinito , pois ela e ilimitada

superiormente, de c.p(n) ≥ s(n), p(n) ≥ s(n)

centao p(n) tambem e ilimitada su-

periormente e ainda por ser crescente tem limite infinito, logo a serie associada

p(n) =n∑

k=n0

bk tende a infinito.

Z Exemplo 7. Mostrar que∞∑k=1

kk = ∞.

De 1 < k elevamos a k, 1 < kk aplicamos a soman∑

k=1

n =n∑

k=1

1 <n∑

k=1

kk

por comparacao (como sao series de termos positivos) segue que∞∑k=1

kk = ∞.

Z Exemplo 8. Se 0 < c e 1 < |a| entao∑ 1

c+ akconverge.

Vale1

c+ ak<

1

ake a segunda serie converge, logo por comparacao a primeira converge.

Vamos usar o seguinte pequeno resultado em certas demonstracoes.

b Propriedade 11. Sejam (xn) e (yn) sequencias, se ∆xn = ∆yn para todo n, entao

xn = yn + c para alguma constante c.

ê Demonstracao. Aplicamos o somatorion−1∑k=1

em cada lado na igualdade ∆xk =

∆yk e usamos a soma telescopica, de onde segue

xn − x1 = yn − y1 ⇒ xn = yn + x1 − y1︸ ︷︷ ︸=c

.

$ Corolario 2. Se ∆xn = ∆yn ∀ n e existe t ∈ N tal que xt = yt entao xn = yn para

todo n. Tal propriedade vale pois xn = yn + c, tomando n = t segue xt = yt + c que

implica c = 0, logo xn = yn para todo n.

b Propriedade 12. Seja n > 0 ∈ N entao

n−1∑s=0

2s+1−1∑k=2s

f(k) =2n−1∑k=1

f(k).

Page 15: Series

CAPITULO 1. SERIES 14

ê Demonstracao.[1-Soma telescopica]

n−1∑s=0

2s+1−1∑k=2s

f(k) =n−1∑s=0

[2s+1−1∑k=0

f(k)−2s−1∑k=0

f(k)︸ ︷︷ ︸g(s)

] =n−1∑s=0

∆g(s) = g(n)− g(0)︸︷︷︸=0

=2n−1∑k=1

f(k).

ê Demonstracao.[2] Para n = 1

0∑s=0

2s+1−1∑k=2s

f(k) =2−1∑k=20

f(k) =21−1∑k=1

f(k)

Temos que

∆n−1∑s=0

2s+1−1∑k=2s

f(k) =2n+1−1∑k=2n

f(k)

e

∆2n−1∑k=1

f(k) =2n+1−1∑k=1

f(k)−2n−1∑k=1

1

kr=

2n+1−1∑k=2n

f(k) +2n−1∑k=1

f(k)−2n−1∑k=1

f(k) =2n+1−1∑k=2n

f(k).

logo esta provada a igualdade.

1.2.4 Criterio de condensacao de Cauchy

b Propriedade 13 (Criterio de condensacao de Cauchy). Seja (xn) uma sequencia

decrescente de termos positivos entao∑

xk converge ⇔∑

2k.x2k converge.

ê Demonstracao. Usaremos a identidade

n−1∑s=0

2s+1−1∑k=2s

f(k) =2n−1∑k=1

f(k).

⇒).

Vamos provar que se∑

xk converge entao∑

2k.x2k converge, usando a contraposi-

tiva, que e equivalente logicamente, vamos mostrar que se∑

2k.x2k diverge entao∑

xk

diverge.

Como xk e decrescente entao vale

2sx2s+1 =2s+1−1∑k=2s

x2s+1 ≤2s+1−1∑k=2s

xk

Page 16: Series

CAPITULO 1. SERIES 15

aplicandon−1∑s=0

segue

1

2

n−1∑s=0

2s+1x2s+1 ≤2n−1∑k=1

xk

logo se∑

2sx2s diverge entao∑

xk diverge.

⇐).

Vamos provar que se∑

2k.x2k converge entao entao∑

xk converge, de maneira

direta. Usando que2s+1−1∑k=2s

xk ≤2s+1−1∑k=2s

x2s = 2sx2s

aplicandon−1∑s=0

segue que

2n−1∑k=1

xk ≤n−1∑s=0

2sx2s

daı se∑

2sx2s converge entao∑

xk converge .

Z Exemplo 9. A serie∞∑k=3

1

[ln(k)]s

diverge para qualquer valor real de s. Se s ≤ 0 o resultado vale pois temos serie com

soma de [ln(k)]−s que nao converge para zero, se s > 0 temos que ln(k + 1) > ln(k) logo

[ln(k + 1)]s > [ln(k)]s e daı

1

[ln(k)]s>

1

[ln(k + 1)]s

entao a sequencia e decrescente de termos positivos e podemos aplica o criterio de con-

densacao de Cauchy∞∑k=3

2k

[k]s[ln(2)]s

tal serie diverge, pois o termo geral nao tende a zero.

1.2.5 Series do tipo∞∑k=1

1

kpe divergencia da serie harmonica.

b Propriedade 14. A serie∞∑k=1

1

kpconverge se p > 1 e diverge se p ≤ 1.

Page 17: Series

CAPITULO 1. SERIES 16

ê Demonstracao. Pelo criterio de condensacao de Cauchy a serie∞∑k=1

1

kpconverge,

se e somente se,∞∑k=1

2k

2kp=

∞∑k=1

2k(1−p) , tal serie geometrica converge se 1− p < 0, isto e,

p > 1 e diverge caso 1− p ≥ 0 ⇒ p ≤ 1.

Z Exemplo 10. Estudar a convergencia da serie

∞∑k=1

(√k + 1−

√k)p.

Primeiro racionalizamos o termo somado

√k + 1−

√k =

(√k + 1−

√k)(

√k + 1 +

√k)

√k + 1 +

√k

=k + 1− k

√k + 1 +

√k=

1√k + 1 +

√k,

√k ≤

√k + 1 ⇒ 2

√k ≤

√k + 1 +

√k ⇒ 1

√k + 1 +

√k≤ 1

2√k,

elevando a p segue que

(1

√k + 1 +

√k)p ≤ 1

2pkp2

por comparacao sep

2> 1 ⇔ p > 2, a serie converge . De maneira similar

1

2p(k + 1)p2

≤ (1

√k + 1 +

√k)p,

por comparacao diverge casop

2≤ 1.

Z Exemplo 11 (IME-1964). Estude a convergencia das series.

1.∞∑k=1

13√k.

2.∞∑k=1

1

ek.

3.∞∑k=1

ln(k)

k.

1. A primeira serie diverge pois∞∑k=1

1

k13

e uma serie do tipo∞∑k=1

1

kp, com p =

1

3< 1,

que vimos ser divergente.

Page 18: Series

CAPITULO 1. SERIES 17

2. A serie∞∑k=1

1

ek, converge por ser serie geometrica com 0 <

1

e< 1.

3. A serie∞∑k=1

ln(k)

kdiverge pois para k grande vale ln(k) > 1, daı

ln(k)

k>

1

k, como

∞∑k=1

1

kdiverge, entao por comparacao

∞∑k=1

ln(k)

ktambem diverge.

Z Exemplo 12. Calcular o limite

limn→∞

n∑k=0

1

(n+ k)r

para r > 1 real. Escrevemos o somatorio como

n∑k=0

1

(n+ k)r=

2n∑k=n

1

(k)r=

2n∑k=1

1

(k)r−

n−1∑k=1

1

(k)r

com r > 1 cada uma das series limn−1∑k=1

1

(k)r= s e lim

2n∑k=1

1

(k)r= s convergem e para o

mesmo valor, como a diferenca dos limites e o limite da diferenca em sequencias conver-

gentes, segue que

limn→∞

n∑k=0

1

(n+ k)r= lim(

2n∑k=1

1

(k)r−

n−1∑k=1

1

(k)r) = lim

2n∑k=1

1

(k)r− lim

n−1∑k=1

1

(k)r= s− s = 0.

b Propriedade 15. Se ak ≥ 0 ∀k ∈ N e (a′k) e uma subsequencia de (ak) entao∞∑k=c

ak <∞ implica que∞∑k=c

a′k <∞.

ê Demonstracao. Seja N1 o conjunto dos ındices da subsequencia (a′k), definimos

ck = ak se k ∈ N1 e ck = 0 se k /∈ N1 para todo k natural, entao temos que ck ≤ ak pois

caso k ∈ N1 temos ck = ak caso k /∈ N1 ck = 0 ≤ ak logo em qualquer caso vale ck ≤ ak,

tomando a soma em ambos lados temos

g(n) =n∑

k=c

ck ≤n∑

k=c

ak <∞

logo a soma dos termos da subsequencia g(n) e limitada superiormente e temos tambem

∆g(n) = cn+1 ≥ 0 pois se cn+1 = 0 vale cn+1 ≥ 0 e se cn+1 = an+1 e por propriedade da

Page 19: Series

CAPITULO 1. SERIES 18

sequencia (an) temos an+1 ≥ 0 de onde segue cn+1 = an+1 ≥ 0, entao a sequencia g(n) e

limitada superiormente e nao-decrescente logo convergente e vale∞∑k=c

ck <∞.

Mostramos entao que se (an) e uma sequencia tal que an ≥ 0 e a serie dos seus termos

converge entao dada qualquer subsequencia de de (a′n) de (an) entao a serie dos termos

dessa subsequencia tambem converge.

1.2.6 Divergencia da serie harmonica.

Z Exemplo 13 (Serie Harmonica). Os numeros harmonicos sao definidos como

Hn =n∑

k=1

1

k

temos que lim1

n= 0 satisfaz a condicao necessaria para convergencia de series mas vamos

mostrar que a serie

limHn =∞∑k=1

1

k= ∞

, isto e, a serie diverge.

Suponha que a serie harmonica seja convergente, denotando limHn = H Sejam N1 o

subconjunto de N dos ındices pares e N2 o conjunto dos numeros ımpares. Se Hn converge

temos que a serie sobre suas subsequencias tambem converge, sendo entao

n∑k=1

1

2k − 1= tn,

∞∑k=1

1

2k − 1= t

n∑k=1

1

2k= sn,

∞∑k=1

1

2k= s =

1

2

∞∑k=1

1

k=H

2

temos H2n = sn + tn tomando o limite limH2n = H = lim(sn + tn) = s+ t , como s =H

2

segue que t =H

2pois a soma deve ser H, desse modo a diferenca t− s = 0, mas

tn − sn =n∑

k=1

1

2k − 1−

n∑k=1

1

2k=

n∑k=1

1

(2k)(2k − 1)=

1

2+

n∑k=2

1

(2k)(2k − 1)> 0

logo

lim tn − sn = t− s > 0

de onde segue t > s que e absurdo. Pode-se mostrar que lim tn − sn = ln(2).

Page 20: Series

CAPITULO 1. SERIES 19

Z Exemplo 14. Na serie harmonica percebemos que

1

3+

1

4>

2

4=

1

2

1

5+

1

6+

1

7+

1

8>

4

8=

1

21

9+

1

10+

1

11+

1

12+

1

13+

1

14+

1

15+

1

16>

8

16=

1

2

podemos continuar agrupando os termos das somas dessa maneira, vendo que a soma dos

termos harmonicos nao sao limitados superiormente.

Usando o criterio de condensacao de Cauchy∞∑k=1

2k

2k=∑

1 diverge.

1.2.7 Divergencia de∞∑k=1

1

kpcom p < 1.

$ Corolario 3.∞∑k=1

1

kpdiverge se p < 1. Para p < 1 vale kp < k e daı

1

k<

1

kp, daı por

comparacao como∞∑k=1

1

kdiverge isso implica que

∞∑k=1

1

kptambem diverge.

Z Exemplo 15. A serie∞∑k=0

k√12

k + 3diverge, pois vale que

k√12

k + 3>

1

k + 3, onde a serie

da segunda diverge.

b Propriedade 16. A serie∞∑k=2

1

k(ln(k) + c)r

diverge se r ≤ 1 e converge se r > 1. c ≥ 0.

ê Demonstracao.

Usamos o criterio de condensacao de Cauchy∑ 2k

2k(ln(2k) + c)r=∑ 1

(k ln(2) + c)r

que diverge se r ≤ 1 e converge se r > 1 .

$ Corolario 4. A seguinte serie converge

∞∑k=2

1

[ln(k)]k.

Page 21: Series

CAPITULO 1. SERIES 20

Como ln(k) > 2 para k suficientemente grande , tem-se [ln(k)]k > 2k ⇒

1

[ln(k)]k<

1

2k,

logo por criterio de comparacao∞∑k=2

1

[ln(k)]kconverge .

Z Exemplo 16. Estudar a convergencia da serie

∞∑k=1

1

kHk

Hn =n∑

k=1

1

k.

podemos mostrar que Hn ≤ 1 + ln(n) daı nHn ≤ n(1 + ln(n))1

n(ln(n) + 1)≤ 1

nHn

,

logo a primeira diverge por criterio de comparacao .

Z Exemplo 17. A serie∞∑k=2

1

k ln(k)(ln(ln k))r

diverge se r ≤ 1 e converge se r > 1.

Aplicamos o metodo de condensacao de cauchy

∞∑k=2

2k

2k ln(2k)(ln(ln 2k))r=

∞∑k=2

1

k ln(2)(ln(k) + ln((ln 2)))r

que converge se r > 1 e diverge se r ≤ 1.

Z Exemplo 18. Provar que a serie∑ ln(n)

n2converge. Pelo criterio de condensacao

de Cauchy temos que ∑ 2n ln(2n)

2n.2n=∑ n ln(2)

2n

tal serie converge, logo a primeira tambem converge.

Z Exemplo 19. Mostrar que a serie∞∑k=1

1

k2converge, usando o criterio de comparacao.

Comecaremos com o somatorio

n∑k=2

1

k(k − 1)= −

n∑k=2

1

k− 1

k − 1= − 1

k − 1

∣∣∣∣n+1

2

== − 1

n+ 1 =

n− 1

n

Page 22: Series

CAPITULO 1. SERIES 21

onde usamos soma telescopicab∑

k=a

∆f(k)︸ ︷︷ ︸=f(k+1)−f(k)

= f(b + 1) − f(a) = f(k)

∣∣∣∣b+1

a

, ∆f(k) =

f(k+1)−f(k) e apenas uma notacao para essa diferenca. Tomando o limite na expressao

acima

lim− 1

n+ 1 = 1 =

∞∑k=2

1

k(k − 1).

Vamos mostrar com esse resultado que a serie∞∑k=1

1

k2converge , temos que para k > 1

1

k(k − 1)>

1

k2

pois

k2 > k2 − k

k > 0

e k > 1 por analise de sinal , logo aplicando o somatorio∞∑k=2

1

k(k − 1)>

∞∑k=2

1

k2

somando 1 em ambos lados e usando o resultado da serie que foi calculada

2 > 1 +∞∑k=2

1

k2=

∞∑k=1

1

k2.

Z Exemplo 20. Exemplo de sequencia x(n) que diverge, porem, ∆x(n) converge

para zero. Sabemos que uma condicao necessaria mas nao suficiente para convergencia

de uma serie∞∑k=1

f(k) e que lim f(k) = 0, porem nao e suficiente pois existem series em

que lim f(k) = 0 e a serie diverge, um exemplo desse tipo de serie e a serie harmonica, se

temos lim f(k) = 0 e a sequencias x(n) =n∑

k=1

f(k) diverge, temos que ∆x(n) = f(n+ 1)

cujo limite lim∆x(n) = f(n + 1) = 0 , no caso especial x(n) =n∑

k=1

1

kdiverge, porem

∆x(n) =1

n+ 1converge para zero.

b Propriedade 17. Seja

g(n) =n∑

k=a

f(k)

entao limf(k) = 0 equivale a lim∆g(n) = 0.

Page 23: Series

CAPITULO 1. SERIES 22

ê Demonstracao. Temos que ∆g(n) = f(n + 1) logo se limf(k) = 0 temos

lim∆g(n) = 0 e se lim∆g(n) = 0 implica limf(k) = 0 .

b Propriedade 18. Sejam∞∑

n=u

an e∞∑n=s

bn series de termos positivos. Se∞∑n=s

bn = ∞ e

existe n0 ∈ N tal quean+1

an≥ bn+1

bnpara todo n > n0 entao

∞∑n=u

an = ∞.

ê Demonstracao.an+1

an≥ bn+1

bn, Qak ≥ Qbk tomando o produtorio com k variando

de k = n0 + 1 ate n− 1 na desigualdade em ambos lados segue

n−1∏k=n0+1

Qak =anan0+1

≥n−1∏

k=n0+1

Qbk =bnbn0+1

, an ≥ an0+1

bn0+1

bn

pois temos termos positivos, tomando a serie temos

∞∑n=n0+1

an ≥ an0

bn0

∞∑n=n0+1

bn = ∞

logo a serie tende ao infinito por comparacao.

Z Exemplo 21. Mostre que a sequencia definida por

f(n) =n∑

k=1

1

k + n

converge para um numero em [0, 1]. Primeiro vamos mostrar que a sequencia e crescente

f(n+ 1)− f(n) =n+1∑k=1

1

k + n+ 1−

n∑k=1

1

k + n=

1

2(n+ 1)+

n∑k=1

1

k + n+ 1−

n∑k=1

1

k + n=

=1

2(n+ 1)+

n∑k=1

(1

k + n+ 1− 1

k + n

)=

1

2(n+ 1)+

n∑k=1

∆1

k + n=

=1

2(n+ 1)+

1

2n+ 1− 1

n+ 1=

1

2n+ 1− 1

2(n+ 1)

mas temos1

2n+ 1− 1

2(n+ 1)> 0 pois

1

2n+ 1>

1

2(n+ 1), 2n+ 2 > 2n+ 1, 2 > 1 agora

vamos mostrar que a serie e limitada superiormente por 1 temos

1

n>

1

k + n

Page 24: Series

CAPITULO 1. SERIES 23

k + n > n

pois nosso valor k e maior que zero, tomando o somatorio em ambos lados com k em [1, n]

temosn∑

k=1

1

n= n

1

n= 1 >

n∑k=1

1

k + n

assim a serie e limitada superiormente , crescente e limitada inferiormente pelo seu pri-

meiro termo1∑

k=1

1

k + 1=

1

2

logo a sequencia assume valores no intervalo [0, 1]. O limite dessa sequencia e ln(2),

podemos mostrar isso transformando o limite numa integral2 podemos usar tambem a

funcao digamma

∆ψ(k + n) =1

k + nn∑

k=1

∆ψ(k + n) = ψ(2n+ 1)− ψ(n+ 1) =n∑

k=1

1

k + n= H2n −Hn =

2n∑k=1

(−1)k+1

k

tendo limite3 ln(2).

Z Exemplo 22. Mostre que a serie∞∑k=0

1

kke convergente.

Para k > 2 vale kk > k2 daı1

kk<

1

k2, da convergencia de

∑ 1

k2segue a convergencia

de∞∑k=0

1

kk.

Z Exemplo 23. A serie∞∑k=0

1

kln(k)e convergente pois para k grande temos ln(k) > 2

daı segue1

kln(k)<

1

k2.

1.2.8 Series de funcoes racionais

Vamos estudar convergencia de series do tipo∑ p(x)

g(x)onde p(x) e g(x) = 0 sao

polinomios.

2Resolvido no texto series 23Resolvido em funcoes especiais

Page 25: Series

CAPITULO 1. SERIES 24

b Propriedade 19. Sejam polinomios

p∑k=0

akxk ,

p+1∑k=0

bkxk e c >

bp+1

apcom c > 0, entao

existe x0 ∈ R tal que x > x0 implica

1

cx<

p∑k=0

akxk

p+1∑k=0

bkxk

Z Exemplo 24. Estudar a convergencia da serie

∞∑n=1

n∏s=1

(p+ s)

(q + s)

com p, q reais em [0,∞)

Para q ≤ p temos q + s ≤ p + s ⇒ 1 ≤ p+ s

q + slogo 1 ≤

n∏s=1

(p+ s)

(q + s)= an portanto an

nao converge para zero e a serie nao pode convergir .

Suponha agora p < q, existe t real tal que p+ t = q o termo an se escreve como

n∏s=1

(p+ s)

(p+ t+ s).

Vamos analisar os casos de t ≤ 1 e 2 ≤ t, no primeiro

p+ t+ s ≤ p+ s+ 1 ⇒ p+ s

p+ s+ 1≤ p+ s

p+ t+ s

aplicandon∏

s=1

na desigualdade acima temos um produto telescopico

n∏s=1

p+ s

p+ s+ 1=

p+ 1

p+ n+ 1≤

n∏s=1

p+ s

p+ t+ s

por comparacao com serie harmonica a soma de an diverge nesse caso . Sendo agora

2 ≤ t

2 ≤ t⇒ p+ s+ 2 ≤ p+ s+ t⇒ p+ s

p+ s+ t≤ p+ s

p+ s+ 2

aplicandon∏

s=1

, novamente temos um produto telescopico

n∏s=1

p+ s

p+ s+ t≤

n∏s=1

p+ s

p+ s+ 2=

2∏s=1

p+ s

p+ s+ n=

(p+ 1)(p+ 2)

(p+ 1 + n)(p+ 2 + n)

logo por criterio de comparacao a soma de an converge .

Page 26: Series

CAPITULO 1. SERIES 25

1.2.9 Criterio de Cauchy para series

b Propriedade 20 (Criterio de Cauchy para series). Tem-se que uma sequencia e

convergente em R ⇔ ela e de Cauchy, logo se definimos s(n) =n∑

k=a

ak temos que a serie

e convergente ⇔ para cada ε > 0 existe n0 ∈ N tal que n > n0 e para todo p ∈ N vale

|s(n+ p)− s(n)| < ε temos que

s(n+ p)− s(n) =

n+p∑k=a

ak −n∑

k=a

ak =

n+p∑k=n+1

ak +n∑

k=a

ak −n∑

k=a

ak =

=

n+p∑k=n+1

ak

logo temos que ter

|n+p∑

k=n+1

ak| < ε

b Propriedade 21. Se f(k) ≥ 0 para k ∈ [a,∞)Z , a ∈ Z entao a serie∞∑k=a

f(k) e

convergente ou diverge para +∞.

ê Demonstracao. Definindo g(n) =n∑

k=a

f(k) temos que ∆g(n) = f(n+1) ≥ 0 logo

g(n) e uma sequencia nao decrescente, se g(n) for limitada entao g(n) converge implicando

que a serie converge, se g(n) nao for limitada, por ser nao decrescente ela diverge para

+∞, implicando que a serie diverge para +∞.

Da mesma maneira tem-se

b Propriedade 22. Se4 f(k) ≤ 0 para k ∈ [a,∞)Z , a ∈ Z entao a serie∞∑k=a

f(k) e

convergente ou diverge para −∞.

ê Demonstracao. Definindo g(n) =n∑

k=a

f(k) temos que ∆g(n) = f(n+1) ≤ 0 logo

g(n) e uma sequencia nao crescente, se g(n) for limitada entao g(n) converge implicando

que a serie converge, se g(n) nao for limitada, por ser nao crescente ela diverge para −∞,

implicando que a serie diverge para −∞.

4A demonstracao e a mesma que da propriedade anterior, apenas mudando ≥ por ≤, +∞ por −∞ e

nao decrescente por nao crescente

Page 27: Series

CAPITULO 1. SERIES 26

b Propriedade 23.∞∑k=a

b converge ⇔ b = 0.

ê Demonstracao. Seja g(n) =n∑

k=a

b, Se b = 0 temos g(n) =n∑

k=a

0 = 0 assim temos

que serie e o limite da sequencia constante 0, lim g(n) = 0 =∞∑k=a

0. Se∞∑k=a

b converge, pela

condicao necessaria para convergencia temos que ter lim b = 0, como b e constante, por

propriedade de limites tem-se lim b = b lim 1 = b = 0 logo b tem que ser zero para que a

serie seja convergente

∞∑k=a

0 = 0 = 0 + 0 + 0 + 0 + 0 + 0 + · · · = 0.

Se b = 0 entao a serie diverge para +∞ ou−∞ pois ∆g(n) = b, se b > 0 tem-se que a serie e

crescente e divergente logo seu limite e +∞ se b < 0 segue que ∆g(n) < 0 logo decrescente

e divergente assim seu limite e −∞. As reduzidas sao dadas porn∑

k=a

b = b(n+ 1− a), se

b < 0∞∑k=a

b = b+ b+ b+ b+ · · · = −∞

se b > 0∞∑k=a

b = b+ b+ b+ b+ · · · = +∞.

b Propriedade 24 (A serie harmonica diverge para +∞.). Seja 2n ≥ k (n, k naturais

maiores que zero), entao

1

k≥ 1

2n

tomando a soma em [n+ 1, 2n]

2n∑k=n+1

1

k≥

2n∑k=n+1

1

2n=

n

2n=

1

2

assim o criterio de Cauchy, nao e valido para serie harmonica, pois tomando ε =1

2e

p = n nao temos

|2n∑

k=n+1

1

k| < 1

2

Page 28: Series

CAPITULO 1. SERIES 27

para n suficientemente grande, pois vale para qualquer n

2n∑k=n+1

1

k≥ 1

2

sendo g(n) =n∑

k=1

1

ktemos ∆g(n) =

1

n+ 1> 0 logo a serie e crescente. Sendo crescente e

divergente entao ela tem limite +∞. Com a serie harmonica temos um exemplo de serie

cujo termo somado tem limite 0 porem a serie diverge lim1

n= 0.

1.3 Series absolutamente convergentes

m Definicao 3 (Series absolutamente convergentes). Uma serie∞∑k=a

ak e dita absoluta-

mente convergente se∞∑k=a

|ak| e convergente.

m Definicao 4 (Serie condicionalmente convergente). Uma serie∞∑k=a

ak e condicional-

mente convergente se∞∑k=a

ak converge porem∞∑k=a

|ak| diverge.

1.3.1 Toda serie absolutamente convergente e convergente

⋆ Teorema 1 (Toda serie absolutamente convergente e convergente). Se s(n) =n∑

k=b

|ak|

converge entaon∑

k=b

ak converge.

ê Demonstracao. Sen∑

k=b

|ak| converge, podemos usar o criterio de Cauchy, que

garante que para todo ε > 0 existe n0 ∈ N , tal que para n > n0 e p ∈ N vale

|n+p∑

k=n+1

|ak| | =n+p∑

k=n+1

|ak| < ε

mas vale a desigualdade

|n+p∑

k=n+1

ak| ≤n+p∑

k=n+1

|ak| < ε

logon∑

k=b

ak e uma sequencia de Cauchy, portanto converge.

Page 29: Series

CAPITULO 1. SERIES 28

ê Demonstracao.[2] Temos5 que ak ≤ |ak| e tambem −ak ≤ |ak| logo dessa ultima

0 ≤ ak+|ak| que por sua vez e menor que 2|ak|, como 2∞∑k=0

|ak| converge entao∞∑k=0

ak+|ak|

tambem converge por comparacao, daı

∞∑k=0

ak + |ak| −∞∑k=0

|ak| =∞∑k=0

ak

converge por ser soma de series convergentes.

Daremos uma outra demonstracao dessa propriedade usando o conceito de parte po-

sitiva e negativa de uma serie.

1.3.2 Parte negativa e positiva de uma serie

m Definicao 5 (Parte negativa e positiva de uma serie). Seja∑

ak uma serie, para

cada k definimos a parte positiva pk da seguinte maneira

pk =

ak se ak ≥ 0

0 se ak < 0

Definimos a parte negativa qk como

qk =

−ak se ak ≤ 0

0 se ak > 0

b Propriedade 25. Valem

1. an = pn − qn

2. |an| = pn + qn.

ê Demonstracao.

1. Se an ≥ 0 entao qn = 0, an = pn, se an < 0 entao pn = 0 e an = −qn.

2. Se an ≥ 0 entao |an| = an = pn pois qn = 0. Se an < 0 entao |an| = −an = qn pois

pn = 0.

5Solucao de Diogenes Mota

Page 30: Series

CAPITULO 1. SERIES 29

$ Corolario 5. Vale que pn ≤ |an| e qn ≤ |an|, pois segue da relacao |an| = pn + qn e do

fato de ambos serem nao negativos.

$ Corolario 6. Se∑

|an| e convergente entao∑

an e convergente. Vale que pn ≤

|an| e qn ≤ |an|, daı∑

pn e∑

qn sao convergentes por criterio de comparacao, daı∑(pn − qn) =

∑pn −

∑qn e convergente logo

∑an e convergente.

b Propriedade 26. Se uma serie∑

an e condicionalmente convergente entao∑

pn =

∞ =∑

qn.

ê Demonstracao. Vale quen∑

k=b

ak =n∑

k=b

pk −n∑

k=b

qk se∑

pn fosse convergente,

entao∑

qn tambem o seria, logo∑

|an| =∑

pn +∑

qn seria convergente o que

contradiz a hipotese.

$ Corolario 7. Seja∞∑k=b

ak uma serie absolutamente convergente entao∞∑k=b

ak(−1)yk e

convergente onde (yk) e uma sequencia qualquer de numeros naturais. A propriedade vale

pois∞∑k=b

|ak(−1)yk | =∞∑k=b

|ak| entao∞∑k=b

ak(−1)yk e absolutamente convergente portanto

convergente.

Esse resultado diz que se tomamos uma serie absolutamente convergente e trocamos

os sinais dos termos da serie de maneira arbitraria entao ainda assim a serie continua

sendo convergente.

Z Exemplo 25. Analisar a convergencia da serie∞∑k=1

senka

kr

onde r > 1, real. Vale sempre |senka| ≤ 1, daı|senka|kr

≤ 1

krpor comparacao temos

∞∑k=1

|senka|kr

≤∞∑k=1

1

kr

a serie da direita converge, logo a serie∞∑k=1

senka

kre absolutamente convergente, e conver-

gente. O mesmo argumento pode ser feito para mostrar que∞∑k=1

coska

kr

Page 31: Series

CAPITULO 1. SERIES 30

e absolutamente convergente.

$ Corolario 8. Seja∞∑k=d

bk uma serie convergente, com bk ≥ 0 para todo k ∈ Z. Se

existem c > 0 e n0 ∈ N tais que |ak| ≤ c.bk para todo k > n0, entao a serie∞∑k=d

ak

e absolutamente convergente. Tal propriedade vale pois podemos aplicar o criterio de

comparacao de series para concluir que∞∑k=d

|ak| converge, logo∞∑k=d

ak sera absolutamente

convergente.

$ Corolario 9. Se para todo k > n0, tem-se |ak| ≤ c.bk onde 0 < b < 1 e c > 0, entao a

serie∞∑k=d

ak e absolutamente convergente. Pois a serie∞∑k=d

bk e convergente pela condicao

0 < b < 1.

b Propriedade 27. Sejam an ≥ 0 e∑

an convergente, entao∑

anxn e absolutamente

convergente ∀ x ∈ [−1, 1].

ê Demonstracao. Com x ∈ [−1, 1] vale |x| ≤ 1 daı∑|anxn| =

∑an|x|n ≤

∑an

logo∑

anxn e absolutamente convergente.

b Propriedade 28. Se∑

an e convergente com an ≥ 0 e x ∈ R arbitrario entao∑ansen(nx) e

∑ancos(nx) sao absolutamente convergentes.

ê Demonstracao. Vale que∑|ansen(nx)| =

∑an|sen(nx)| ≤

∑an∑

|ancos(nx)| =∑

an|cos(nx)| ≤∑

an

logo ambas series convergem absolutamente.

b Propriedade 29. Se∑

ak e absolutamente convergente e lim bn = 0 entao cn =n∑

k=1

akbn−k → 0.

Page 32: Series

CAPITULO 1. SERIES 31

ê Demonstracao. Existe B > 0 tal que |bn| < B, ∀ n ∈ N. Vale∞∑k=1

|ak| = A.

Dado ε > 0 existe n0 ∈ N tal que n > n0 implica |bn| <ε

2Ae por

n∑k=1

|ak| ser de cauchy

vale |n∑

k=n0+1

ak| <ε

2Bentao para n > 2n0 (n− n0 > n0) segue que

|n∑

k=1

akbn−k| ≤n∑

k=1

|ak||bn−k| =n0∑k=1

|ak||bn−k|+n∑

k=n0+1

|ak||bn−k| ≤

≤n0∑k=1

|ak|ε

2A+

n∑k=n0+1

|ak|B ≤ Aε

2A+εB

2B≤ ε

2+ε

2= ε

isso implica que lim cn = 0.

b Propriedade 30. Seja∑

an uma serie qualquer, denotamos

S = {∑k∈A

ak, tal que A e qualquer conjunto finito de ındices de (ak)}.

∑ak e absolutamente convergente ⇔ S e limitado.

ê Demonstracao. ⇒ Se∑

ak e absolutamente convergente entao a soma dos

termos positivos e no maximo p =∑

pk e a soma dos termos negativos e no maximo

−q = −∑

qk, logo S e um conjunto limitado, pois qualquer outra combinacao de soma

de termos positivos e negativos do conjunto deve estar entre esses dois valores. ⇐. Se

S e limitado entao∑

pn e∑

qn sao limitados e por isso convergentes pois determinam

sequencias nao-decrescentes limitadas superiormente, daı segue que∑

|an| =∑

pn +∑qn e convergente.

1.3.3 Teste da raiz-Cauchy

b Propriedade 31 (Teste da raiz). Se existe b tal que k√

|ak| ≤ b < 1 para todo k > n0

entao∞∑k=d

ak e absolutamente convergente. Se lim sup k√

|ak| = b < 1 temos essa condicao,

pois o lim sup e o maior dos valores de aderencia da sequencia, se houvesse uma infinidade

de valores da raiz em que k√|ak| > b terıamos o lim sup maior que b, entao temos apenas

um numero finito de ındices em que a condicao colocada nao vale.

Page 33: Series

CAPITULO 1. SERIES 32

ê Demonstracao. Temos |ak| ≤ bk para k > n0, logo por criterio de comparacao

segue o resultado, pois ≤ b < 1 entao a serie∑

bk converge e daı tambem∑

|ak| porcriterio de comparacao .

$ Corolario 10 (Teste da Raiz de Cauchy). Se lim n√

|an| < 1 entao∞∑k=d

ak e absolu-

tamente convergente. Pois se lim n√

|an| < 1 existe n0 ∈ N tal que para n > n0 implica

n√|an| < 1.

Se lim n√|an| > 1 entao a serie

∑ak diverge, pois existe n0 ∈ N tal que para k > n0

tem-se 1 < t < a− ε < k√

|ak| < a+ ε logo tn < |ak| o que implica que lim ak = 0 entao a

serie∑

ak diverge .

Se lim n√|an| = 1 entao o teste e inconclusivo. Por exemplo

∑ 1

k2converge e

∑ 1

kdiverge .

b Propriedade 32. Se |an|1n > 1 para uma infinidade de indices n entao lim an = 0 e

a serie∑

an diverge. A mesma observacao vale se usamos o conceito de lim sup, isto e,

se lim sup |an|1n = α > 1.

ê Demonstracao. Se lim an = 0 entao existe n0 ∈ N tal que para n > n0 tem-se

|an| <1

2, se |an|

1n ≥ 1 para uma infinidade de indices n, entao existe um ındice n1 > n0

tal que |an1 |1n1 ≥ 1 logo |an1 | ≥ 1 o que entra em contradicao com a suposicao de que

lim an = 0 entao tal propriedade nao vale, de onde segue que a serie∑

an diverge, pois

se ela fosse convergente entao terıamos lim an = 0.

b Propriedade 33. A serie∞∑k=d

kpak

converge quando |a| < 1 e diverge quando |a| ≥ 1, onde p ≥ 0 .

ê Demonstracao. Aplicamos o teste da raiz lim n√|npan| = |a|(lim n

√n)p = |a|

quando |a| < 1 ela entao converge, se |a| ≥ 1 tem-se que o limite limnpan = 0 entao a

serie diverge.

$ Corolario 11. Seja g(k) um polinomio e |a| < 1, entao

∞∑k=d

g(k)ak

Page 34: Series

CAPITULO 1. SERIES 33

converge absolutamente.

Z Exemplo 26. A serie∞∑k=1

ak ln k converge absolutamente quando |a| < 1 pois

|a|k ln k ≤ |a|kk

por comparacao a serie converge.

Z Exemplo 27. A serie∞∑n=2

1

(lnn)nconverge pois lim n

√1

(lnn)n= lim

1

lnn= 0 < 1,

logo a serie converge.

Z Exemplo 28. Estude a convergencia da serie

∞∑k=1

(k

k + 1

)k2

.

Aplicamos o teste da raiz

((k

k + 1

)k2) 1k

=

(k

k + 1

)k

=1

(1 + 1k)k

→ 1

e< 1

pois e > 1 logo a serie converge .

1.3.4 Teste da razao-D’ Alembert

⋆ Teorema 2 (Teste da razao). Sejam∞∑k=d

ak uma serie de termos nao-nulos e∞∑k=d

bk

uma serie convergente com bk > 0 para todo k natural. Se existe n0 ∈ N tal que

|ak+1||ak|

≤ bk+1

bk

para todo k ≥ n0 entao∞∑k=d

ak e absolutamente convergente. O mesmo resulta se consi-

deramos o conceito de lim sup, assim como fizemos no teste da raiz.

ê Demonstracao. Podemos escrever a desigualdade como

Q|ak| ≤ Qbk,

Page 35: Series

CAPITULO 1. SERIES 34

onde Qbk =bk+1

bk, aplicando o produtorio em ambos lados com k ≥ n0 ate n−1 tem-se

por produto telescopico

n−1∏k=n0

Q|ak| =|an||an0|

≤n−1∏k=n0

Qbk =bnbn0

logo vale

|an| ≤ c.bn

onde c =|an0 |bn0

> 0. Entao por criterio de comparacao∞∑k=d

ak e absolutamente convergente.

$ Corolario 12. Se existe uma constante c tal que 0 < c < 1 tomamos bk = ck e temos

a serie∞∑k=d

ck convergente e aindack+1

ck= c. Se vale

|ak+1||ak|

≤ c para todo k ≥ n0 entao

∞∑k=d

ak e absolutamente convergente pela propriedade anterior.

$ Corolario 13 (Teste da razao de D’Alembert). Se lim|an+1||an|

< 1 entao a serie∞∑k=d

ak

e absolutamente convergente. Pois se vale lim|an+1||an|

< 1 existe n0 tal que para n > n0

tem-se|an+1||an|

< 1.

Se lim|an+1||an|

> 1 entao a serie diverge, pois para k > n0 tem-se

1 < t < a+ ε <|ak+1||ak|

aplicandon∏

n0+1

tem-se

tn−n0 |xn0+1| < |xn+1|

logo (xn) nao converge para zero, logo a serie∑

xk diverge.

Caso lim|xn+1||xn|

= 1 entao o teste e inconclusivo. Nesse caso a serie pode divergir,

como em∑

1, pode convergir como∑ 1

k2.

b Propriedade 34. Se an = 0∀ n ∈ N e existe n0 ∈ N tal que para n ≥ n0 tem-se

|an+1||an|

≥ 1 entao∑

an diverge.

Page 36: Series

CAPITULO 1. SERIES 35

ê Demonstracao. Para k > n0 vale|ak+1||ak|

≥ 1 daı aplicandon∏

k=n0

de ambos lados,

segue por produto telescopico que

|an+1|an0

≥ 1 ⇒ |an+1| ≥ |an0 | > 0

logo nao vale que lim an = 0, portanto a serie∑

an diverge.

b Propriedade 35. Para qualquer sequencia (cn), cn > 0, vale que

lim sup(cn)1n ≤ lim sup

cn+1

cn.

ê Demonstracao. Seja α = lim supcn+1

cn, finito, tomamos b > α qualquer, existe

n0 suficientemente grande tal queck+1

ck≤ b para k ≥ n0 tomamos o produto

n0+p−1∏k=n0

em ambos lados pro produto telescopico (os termos se anulam no primeiro termo do

produtorio) temos

cn0 + p︸ ︷︷ ︸n

≤ bpcn0

cn ≤ cn0b−n0bn

daı tomando a raiz n-esima

n√cn ≤ n

√cn0b

−n0︸ ︷︷ ︸→1

b

logo

lim sup n√cn ≤ b

vale para qualquer b > α logo

lim sup n√cn ≤ α.

$ Corolario 14. Se o teste da razao implica convergencia entao o criterio da raiz tambem

lim sup(cn)1n ≤ lim sup

cn+1

cn< 1

se o criterio da raiz implica divergencia entao o criterio da razao tambem

1 < lim sup(cn)1n ≤ lim sup

cn+1

cn.

Page 37: Series

CAPITULO 1. SERIES 36

Z Exemplo 29. A serie∞∑k=1

ak = a + b + a2 + b2 + a3 + b3 + a4 + b4 + · · · definida

como a2k = bk e a2k−1 = ak onde 0 < a < b < 1 converge. O teste de d’Alembert e

inconclusivo pois ∀ k a2ka2k−1

= (b

a)k > 1 pois de a < b segue 1 <

b

a. O teste de Cauchy

funciona pois para ındices pares2n√bn =

√b < 1 e para ındices ımpares 2n−1

√an < 1, logo

vale para todo n, n√

|an| < 1 e o teste de Cauchy implica que∑

an converge. No caso do

teste de d’Alembert, caso fosse a = b seguiria quea2ka2k−1

= (b

a)k = 1, porem a serie seria

convergente pois2n∑k=1

ak =n∑

k=1

a2k +n∑

k=1

a2k−1 =n∑

k=1

ak +n∑

k=1

bk

sendo que a sequencia das reduzidas e convergente logo a serie e convergente, em especial

esse argumento vale para a = b =1

2.

b Propriedade 36. A serie∞∑k=0

xk

k!converge absolutamente para qualquer x ∈ R dado.

ê Demonstracao. Se x = 0 a serie trunca , se nao pelo teste da razao tomamos

xn =xn

n!e daı

xn+1

xn=

xn+1

(n+ 1)!

n!

xn=

x

n+ 1

cujo limite e 0, logo a serie converge absolutamente pelo criterio da razao.

$ Corolario 15. No texto de sequencias tomamos limites da razao de algumas sequencias

que nos permitem concluir que∞∑n=0

an.n!np

nn

converge se 0 < a < e, no caso especial de p = 0 e a = 1 tem-se que

∞∑n=0

n!

nn

converge, tambem converge∞∑n=0

npan

n!

e se a > 1∞∑n=0

np

an.

Page 38: Series

CAPITULO 1. SERIES 37

$ Corolario 16. Por comparacao com∞∑n=0

n!

nnconcluımos que

∞∑n=0

1

nnconverge.

b Propriedade 37. A sequencia de termo (ln(n+ 1)

(n+ 1))n e limitada.

ê Demonstracao.

Para n ≥ 3 vale (n+ 1

n)n < n daı (n+ 1)n < nn+1 tomando o logaritmo n ln(n+ 1) <

(n + 1) ln(n) logoln(n+ 1)

ln(n)<n+ 1

nelevando a n segue que (

ln(n+ 1)

(n+ 1))n < (

n+ 1

n)n,

sendo menor que uma sequencia limitada segue que ela e limitada.

Z Exemplo 30. Mostrar que∑

(ln(n)

n)n e convergente.

Pelo criterio de D’Alembert, temos

(ln(n+ 1)

(n+ 1))n+1(

(n)

ln(n))n =

ln(n+ 1)

n+ 1(ln(n+ 1)

(n+ 1))n(

n

n+ 1)n

o primeiro limite tende a zero, a segunda expressao e limitada e o terceiro limite converge,

entao tal expressao tende a zero.

Pelo criterio de Cauchy,n

√(ln(n)

n)n =

ln(n)

n→ 0 logo a serie converge.

Z Exemplo 31. Estudamos os valores x reais com os quais as series a seguir convergem.

1.∑

nkxn. n√nk|x|n =

n√nk|x| → |x| entao a serie converge com |x| < 1, ela nao

converge se x = 1 ou x = −1 pois nesses casos o limite do termo somado nao tende

a zero.

2.∑

nnxn. n√nn|x|n = n|x| → ∞ se x = 0 ela so converge para x = 0.

3.∑ xn

nn.

n

√|x|nnn

=|x|n

→ 0, logo ela converge independente do valor de x.

4.∑

n!xn. n√n!|x|n =

n√n!|x| → 0, logo ela so converge com x = 0.

5.∑ xn

n2.

n

√|x|nn2

→ |x|, entao e garantida a convergencia com |x| < 1 , com x = 1

ela converge e com x = −1 tambem, pois e absolutamente convergente.

Page 39: Series

CAPITULO 1. SERIES 38

1.3.5 Criterio de Dirichlet

b Propriedade 38 (Criterio de Dirichlet). Sejam s(n) =n∑

k=1

ak uma sequencia limitada,

(bn) uma sequencia decrescente de numeros positivos com lim bn = 0, entao a serie

∞∑k=1

akbk

e convergente.

ê Demonstracao.

Estamos denotando

� ∆f(k) = f(k + 1)− f(k),

� g(k)

]n+1

1

= g(n+ 1)− g(1)

� s(0) =0∑

k=1

ak = 0 chamado de soma vazia.

Usaremos a identidade chamada de soma por partes (de simples demonstracao)

n∑k=1

f(k)∆g(k) = f(k)g(k)

]n+1

1

−n∑

k=1

g(k + 1)∆f(k).

Temos que s(0) =0∑

k=1

ak = 0 por ser soma vazia e s(n− 1) =n−1∑k=1

ak logo

∆s(n− 1) = s(n)− s(n− 1) =n∑

k=1

ak −n−1∑k=1

ak = an

isto e, ∆Sk−1 = ak, vamos usar a regra de soma por partes

n∑k=1

f(k)∆g(k) = f(k)g(k)

∣∣∣∣n+1

1

−n∑

k=1

g(k + 1)∆f(k)

tomando g(k) = sk−1 e f(k) = bk segue

n∑k=1

bk∆sk−1 =n∑

k=1

bkak = bksk−1

∣∣∣∣n+1

1

−n∑

k=1

sk∆bk = bn+1sn − b1s0︸︷︷︸=0

−n∑

k=1

sk∆bk =

Page 40: Series

CAPITULO 1. SERIES 39

= bn+1sn −n∑

k=1

sk∆bk = bn+1sn +n∑

k=1

sk(−∆bk).

Perceba que lim bn+1sn = 0 pois sn e limitada e lim bn = 0. Como bk e decrescente vale

∆bk = bk+1 − bk ≤ 0 ⇒ −∆bk ≥ 0 e a serie∞∑k=1

sk(−∆bk) e absolutamente convergente

pois, como sk e limitada vale |sk| ≤ c > 0 e

n∑k=1

|sk||(−∆bk)| =n∑

k=1

|sk|(−∆bk) ≤n∑

k=1

c(−∆bk) = −c(bn+1 − b1)

logo por comparacaon∑

k=1

sk(−∆bk) e absolutamente convergente, implicando que a serie

∞∑k=1

akbk e convergente.

$ Corolario 17. Sejam s(n) =n∑

k=1

ak uma sequencia limitada, (bn) uma sequencia

crescente de numeros negativos com lim bn = 0, entao a serie

∞∑k=1

akbk

e convergente.

Pois tomando

∞∑k=1

ak(−bk),

(−bk) e uma sequencia de numeros positivos e como vale bk+1 ≥ bk entao −bk ≥ −bk+1

e (−bk) e decrescente e converge para zero, entao podemos aplicar o criterio de Dirichlet

para concluir que −s =∞∑k=1

ak(−bk) logo tambem converge s =∞∑k=1

akbk.

Z Exemplo 32. Dado x fixo, as series

∞∑k=1

senxk

ke

∞∑k=1

coskx

kconvergem.

Pois

n∑k=1

sen(xk) =−cos(xn+ x

2) + cos(x

2)

2sen(x2)

,n∑

k=1

cos(xk) =sen(xn+ x

2)− sen(x

2)

2sen(x2)

Page 41: Series

CAPITULO 1. SERIES 40

sao limitadas e xk =1

ke decrescente, pois ∆xk =

−1

k(k + 1)< 0 com limxn = 0.

b Propriedade 39. Sejam u0 > 0 , (an) positiva

un+1 = un +anun.

un e convergente ⇔∑

an e convergente.

ê Demonstracao.

Primeiro observamos que (un) e positiva pois un+1 = un+anun

, por inducao, segue pois

u0 > 0 e an > 0

⇒).

Usamos o criterio de Dirichlet. De un+1 = un +anun

temos

∞∑k=0

ak =∞∑k=0

uk∆uk

queremos mostrar que a ultima converge. Suponha que c = lim un entao zn = c − un e

sequencia positiva decrescente e lim∆un = 0 entao a serie

∞∑k=0

(c− uk)∆uk = limn∑

k=0

c∆uk︸ ︷︷ ︸converge

−n∑

k=0

uk∆uk

converge, poisn−1∑k=0

∆uk = un − u0 e limitado (pois converge), logo aplicamos o criterio de

Dirichlet. Portanto∞∑k=0

uk∆uk =∞∑k=0

ak converge.

⇐). (Solucao de Alexandre Cezar) Suponha que∞∑k=0

ak converge, vamos mostrar que

(un) converge, ja sabemos que tal sequencia e crescente, vamos mostrar que e limitada.

Suponha que nao seja limitada, entao existe n0 > 0 tal que un > 1 para n ≥ n0

un − u0 =n−1∑k=n0

∆uk =n−1∑k=n0

akuk

≤n−1∑k=n0

ak <∞∑

k=n0

ak

o que e absurdo pois un seria ilimitada e limitada.

b Propriedade 40 (Criterio de Abel). Se∞∑k=1

ak e convergente e (bn) e uma sequencia

decrescente de numeros positivos entao a serie∞∑k=1

akbk e convergente.

Page 42: Series

CAPITULO 1. SERIES 41

ê Demonstracao.

A sequencia (bn) e limitada inferiormente por zero, sendo decrescente ela converge,

seja entao c seu limite, lim bn = c, lim bn − c = 0. (bn − c) e uma sequencia decrescente

com limite 0, temos entao que a serie∞∑ak(bk − c) e convergente e vale

n∑k=1

ak(bk − c) =n∑

k=1

akbk − c

n∑k=1

ak ⇒

n∑k=1

ak(bk − c) + c

n∑k=1

ak =n∑

k=1

akbk

pela ultima identidade vemos que∞∑k=1

akbk e convergente.

1.3.6 Criterio de Leibniz

b Propriedade 41 (Criterio de Leibniz). Se (bn) e uma sequencia decrescente com

lim bn = 0 entao a serie∞∑k=1

(−1)kbk e convergente.

ê Demonstracao.[1] Se (bn) e uma sequencia decrescente com lim bn = 0 entao (bn)

nao o admite termo negativo, pois caso bn0 < 0 entao para n > n0 tem-se bn ≤ bn0 < 0

e o limite da sequencia nao poderia ser zero. Se existe n0 tal que bn0 = 0 entao para

todos n > n0 tem que valer bn = 0, pois ela nao admite termo negativo, entao nao pode

decrescer ainda mais, nesse caso a soma sera uma soma finita, entao resta apenas o caso de

bn > 0 para todo n, nesse caso temos uma sequencia decrescente de termos positivos com

limite zero en∑

k=1

(−1)k e limitada, entao pelo criterio de Dirichlet a serie e convergente.

ê Demonstracao.[2] Seja sn =n∑

k=1

(−1)k+1bk entao

s2n+2 =2n+2∑k=1

(−1)k+1bk =2n∑k=1

(−1)k+1bk − b2n+2 + b2n+1 = s2n + b2n+1 − b2n+2

como (bn) e nao-crescente tem-se que b2n+1 − b2n+2 ≥ 0, daı s2n+2 − s2n ≥ 0, implicando

que (s2n) e nao-decrescente. Da mesma maneira

s2n+1 =2n+1∑k=1

(−1)k+1bk =2n−1∑k=1

(−1)k+1bk + b2n+1 − b2n = s2n−1 + b2n+1 − b2n,

Page 43: Series

CAPITULO 1. SERIES 42

dessa vez como (bn) e nao-crescente, segue que b2n+1 − b2n ≤ 0 logo s2n+1 − s2n−1 ≤ 0 e a

sequencia (s2n−1) e nao-crescente.

Ambas sequencias sao limitadas pois

s2n =2n∑k=1

(−1)k+1bk =2n−1∑k=1

(−1)k+1bk − b2n = s2n−1 − b2n

logo s2n−1 − s2n = bn ≥ 0 ⇒ s2n−1 ≥ s2n,

s1 ≥ s2n−1 ≥ s2n ≥ s2,

as reduzidas de ordem par e ımpar sao monotonas e limitadas logo convergentes lim s2n =

L1, lim s2n−1 = L2, pela identidade s2n−1 − s2n = bn e lim bn = 0 segue na passagem de

limite que

lim s2n−1 − s2n = lim bn = 0 = L2 − L1

logo L1 = L2 e a serie e convergente.

b Propriedade 42. Seja (xn) uma sequencia nao-crescente com limxn = 0 entao a

serie obtida somando p termos com sinais positivos da sequencia (xn) alternando com p

termos negativos alternadamente e convergente.

ê Demonstracao. A serie pode ser escrita como

∞∑t=1

(−1)t+1

p∑k=1

xk+(t−1)p︸ ︷︷ ︸=yt

=∞∑t=1

(−1)t+1yt

Vamos mostrar que essa serie satisfaz os criterio de Leibniz. Como limxn = 0 entao o

limite de qualquer subsequencia de (xn) tambem tende a zero, logo limt→∞

xk+(t−1)p = 0

, para todo k fixo, tem-se lim yt = lim

p∑k=1

xk+(t−1)p = 0. Agora vamos mostrar que a

sequencia (yt) e nao-crescente, como (xn) e nao-crescente temos que xk+tp ≤ xk+(t−1)p

para todo k, aplicando

p∑k=1

tem-se

yt+1 =

p∑k=1

xk+tp ≤p∑

k=1

xk+(t−1)p = yt

daı yt e nao-crescente, logo vale o criterio de Leibniz, implicando que∞∑t=1

(−1)t+1

p∑k=1

xk+(t−1)p

e convergente.

Page 44: Series

CAPITULO 1. SERIES 43

Z Exemplo 33. A serie obtida somando p termos com sinais positivos da sequencia

(xn) = (1

n) alternando com p termos negativos alternadamente e convergente, pois limxn =

0 e xn e decrescente.

Z Exemplo 34. Mostrar que a serie∞∑k=1

(−1)k(k2 + 1)

k3 + 1converge condicionalmente.

Tomando ak =k2 + 1

k3 + 1mostramos que

ak > ak+1,k2 + 1

k3 + 1>

k2 + 2k + 2

k3 + 3k2 + 3k + 2

k5 + 3k4 + 4k3 + 5k2 + 3k + 2 > k5 + 2k4 + 2k3 + k2 + 2k + 2, k4 + 2k3 + 4k2 + k > 0

e temos lim(k2 + 1)

k3 + 1= 0 logo a serie

∞∑k=1

(−1)k(k2 + 1)

k3 + 1converge pelo criterio de Leibniz.

Vamos mostrar agora que∞∑k=1

(k2 + 1)

k3 + 1diverge,

(k2 + 1)

k3 + 1≥ 1

k, k3 + k > k3 + 1, k ≥ 1

logo temos∞∑k=1

(k2 + 1)

k3 + 1>

∞∑k=1

1

k

logo a serie diverge por comparacao.

Z Exemplo 35. A serie∞∑k=1

(−1)k√k

converge por criterio de Leibniz, pois lim1√n= 0

e como√n+ 1 >

√n segue que

1√n>

1√n+ 1

e daı1√n− 1√

n+ 1> 0 a sequencia

e decrescente, podemos aplica o criterio de Leibniz. Observe tambem que a serie dada

pelo quadrado do termo geral diverge pois o termo geral e(−1)2n√n2 =

1

ntermo da serie

harmonica que diverge.

b Propriedade 43. Se∞∑k=0

xk e∞∑k=0

yk convergem e (xk), (yk) sao sequencias de termos

nao negativos, entao∞∑k=0

xk.yk converge.

Page 45: Series

CAPITULO 1. SERIES 44

ê Demonstracao. Sendo f(n) =n∑

k=0

xk.yk, vale ∆f(n) = xn+1.yn+1 ≥ 0, logo a

sequencia das somas parciais e crescente. Vale ainda que a sequencia e limitada superior-

mente poisn∑

k=0

xk.yk ≤ (n∑

k=0

xk)(n∑

k=0

yk).

Z Exemplo 36. A serie∞∑ (−1)k

k + 1converge , pelo criterio de Leibniz temos que

lim1

n+ 1= 0 e (

1

n+ 1) e decrescente. Observamos tambem que essa serie e condicional-

mente convergente pois∞∑ 1

k + 1diverge, pela serie harmonica.

Z Exemplo 37. A serie∞∑k=2

(−1)k

k ln(k)

converge condicionalmente .

Pois1

k ln(k)→ 0, a sequencia e decrescente, logo usamos o criterio de Leibniz, que

implica a serie alternada∞∑k=2

(−1)k

k ln(k)convergir . Tal serie nao converge em modulo6, isto

e,∞∑k=2

1

k ln(k)diverge , portanto

∞∑k=2

(−1)k

k ln(k)

e condicionalmente convergente .

Z Exemplo 38. A serie∞∑k=1

(−1)k√k3 − 6

converge. Pois sequencia de termo xn =1√

n3 − 6

e decrescente

1√n3 − 6

>1√

(n+ 1)3 − 6⇔ (n+ 1)3 − 6 > n3 − 6

que vale e temos limxn = 0. Logo pelo criterio de Leibniz a serie converge.

b Propriedade 44. Seja (xn) tal que xn = 0 para todo n e lim xn = ∞ entao

∞∑k=1

∆xk =∞∑k=1

xk+1 − xk

6Vimos como aplicacao do criterio de Cauchy

Page 46: Series

CAPITULO 1. SERIES 45

diverge e∞∑k=1

∆1

xk=

∞∑k=1

1

xk+1

− 1

xk+1

converge.

ê Demonstracao. A primeira serie tem reduzida

n∑k=1

∆xk = xk

∣∣∣∣n+1

1

= xn+1 − x1

tomando o limite

limn∑

k=1

∆xk = lim xn+1 − x1 = ∞

logo a serie diverge.

A segunda serie tem reduzida

n∑k=1

∆1

xk=

1

xk

∣∣∣∣n+1

1

=1

xn+1

− 1

x1

e tomando o limite mostramos que converge para − 1

x1.

Z Exemplo 39. Seja a serie∞∑k=1

ak(−1)k+1 =2

3− 1

3+2

4− 1

4+2

5− 1

5+2

6− 1

6+ · · · onde

a2k =1

k + 2e a2k−1 =

2

2 + kentao lim ak = 0 e tem termos alternados, porem diverge.

Por que ela nao contradiz o teorema de Leibniz? Tal sequencia nao satisfaz a propriedade

de ser nao-crescente, pois a2k+1 > a2k,2

2 + k + 1>

1

2 + k.

Tal serie realmente diverge pois

2n∑k=1

ak(−1)k+1 =n∑

k=1

a2k−1 −n∑

k=1

a2k =n∑

k=1

2

2 + k− 1

2 + k=

n∑k=1

1

k + 2

que diverge pela divergencia da serie harmonica (perceba acima que separamos os termos

pares dos ımpares na soma).

Z Exemplo 40. Uma serie∑

an pode ser convergente e quando seus termos sao

multiplicados por uma sequencia limitada (xn) a serie∑

anxn, pode divergir, como e o

caso da serie∑ (−1)n

ncom termos multiplicados pela sequencia limitada de termo (−1)n,

gerando a serie∑ 1

nque e divergente. (xn) pode ser convergente e ainda assim

∑anxn

Page 47: Series

CAPITULO 1. SERIES 46

divergir como e o caso de∑ (−1)n√

nque converge pelo criterio de Leibniz e tomando

xn =(−1)n√

n

∑ (−1)n√n

(−1)n√n

=∑ 1

ndiverge.

b Propriedade 45. Se (xn) e limitada e∑

an e absolutamente convergente entao∑anxn e convergente.

ê Demonstracao. Existe m ∈ R tal que |xn| < m ∀ n ∈ N daı |xnan| ≤ m|an| daısegue por comparacao que

∑|xnan| e convergente logo

∑xn.an converge.

1.3.7 Criterio de Kummer

b Propriedade 46 (Criterio de Kummer , parte I). Sejam (xn) com xn > 0 ∀ n ∈ N .

Definindo

f(n) = xnanan+1

− xn+1

com an > 0 ∀ n. Se existe m ∈ N tal que f(n) > α > 0, ∀ n > m, n,m ∈ N entao∞∑k=1

ak

converge.

ê Demonstracao. Seja n > m entao vale

xnanan+1

− xn+1 > α

como an+1 > 0 podemos multiplicar sem alterar a desigualdade

xnan − xn+1an+1 > αan+1 ⇔ −∆(xnan) > αan+1

aplicando a soma com n variando de m+ 1 ate m+ p a desigualdade continua valida

−m+p∑

n=m+1

∆(xnan) = xm+1am+1 − xm+p+1am+p+1 > α

m+p∑n=m+1

an+1 = α

m+p+1∑n=m+2

an

por xm+p+1am+p+1 ser positivo segue

α

m+p+1∑n=m+2

an < xm+1am+1 ⇔m+p+1∑n=m+2

an <xm+1am+1

α

em+p+1∑n=m+2

an =

m+p+1∑n=1

an −m+1∑n=1

an

Page 48: Series

CAPITULO 1. SERIES 47

logom+p+1∑n=1

an −m+1∑n=1

an <xm+1am+1

α⇔

m+p+1∑n=1

an <m+1∑n=1

an +xm+1am+1

α:= K

por m ser fixo, a serie dos termos ak e limitada superiormente e por ser soma de termo

positivos ela converge.

b Propriedade 47 (Criterio de Kummer, parte II). Se existe m ∈ N tal que n > m

implica f(n) ≤ 0 e∞∑k=1

1

xkdiverge, entao

∞∑k=1

ak diverge.

ê Demonstracao. Para k > m Vale

xkakak+1

− xk+1 ≤ 0 ⇔ xkak − xk+1ak+1 = −∆(xkak) ≤ 0

tomando a soma de k = m+ 1 ate n− 1 segue

−n−1∑

k=m+1

∆xkak ≤ 0 ⇔ xm+1am+1︸ ︷︷ ︸=c

≤ xnan ⇔ c

xn≤ an ⇒

∞∑n

c

xn≤

∞∑n

an

a serie∞∑n

an diverge por comparacao com a serie divergente∞∑n

c

xn.

$ Corolario 18. Se lim f(n) > 0 entao a serie∞∑k

ak converge. Pois vai existir m ∈ N

tal que n > m implica f(n) > 0.

Z Exemplo 41. A serie∞∑n=1

n∏s=1

p+ s

q + s

com q, p positivos, converge se q − p > 1 e diverge se q − p ≤ 1.

Com an =n∏

s=1

p+ s

q + stemos

anan+1

=q + n+ 1

p+ n+ 1, tomando xn = p+n+1 > 0, concluımos

quen∑

k=1

1

xkdiverge pela serie harmonica e temos ainda que

(p+ n+ 1)q + n+ 1

p+ n+ 1− p− n− 2 = q + n+ 1− p− n− 2 = q − p− 1

que nao depende de n logo a serie converge para q− p− 1 > 0 ⇒ q− p > 1 e diverge para

q − p− 1 ≤ 0 ⇒ q − p ≤ 1.

Page 49: Series

CAPITULO 1. SERIES 48

m Definicao 6 (Criterio de Raabe). O criterio de Raabe para convergencia de series e

obtido por meio do criterio de Kummer, tomando xn = n. Resumindo o criterio:∞∑k=1

ak

converge se existem α > 0 e m ∈ N tal que vale

nanan+1

− n− 1 = n(anan+1

− 1)− 1 > α

para n > m, em especial se

limn(anan+1

− 1)− 1 = l > 0 ⇔ limn(anan+1

− 1) = s > 1

como∞∑k=1

1

kdiverge, entao se

n(anan+1

− 1) < 1

para n > m entao∞∑k=1

ak diverge o mesmo com limite

limn(anan+1

− 1) = l < 1

entao a serie diverge.

1.4 Comutatividade

m Definicao 7 (Serie comutativamente convergente). Uma serie∑

an e dita ser co-

mutativamente convergente quando para qualquer bijecao f : N → N ( sendo bn = af(n)),

a serie∑

bn e convergente.

A definicao de serie comutativamente convergente tambem funciona para series do tipo∞∑

k=b+1

a′k, pois nesse caso escrevemos a serie como∞∑k=1

a′k+b︸︷︷︸ak

.

$ Corolario 19. Para que∑

an seja comutativamente convergente e necessario que∑an seja convergente, pois f(n) = n e uma bijecao.

m Definicao 8 (Series incondicionalmente convergentes). E uma serie que e comutati-

vamente convergente e toda reordenacao converge para o mesmo limite.

Page 50: Series

CAPITULO 1. SERIES 49

b Propriedade 48. Se∑

|an| converge entao∑

an e comutativamente convergente

e tem-se∑

an =∑

bn onde (bn) e qualquer reordenacao dos termos de (an), (ela e

incondicionalmente convergente.)

ê Demonstracao. Seja∑

akn ,com soma parcial s′n, um rearranjo da serie∑

an.

Dado ε > 0 existe n0 ∈ N tal que m,n ≥ n0 implica

m∑k=n

|ak| ≤ε

2.

Escolhemos p suficientemente grande tal que {1, · · · , n0} e subconjunto da reordenacao

{k1, · · · , kp}. Se n > p + n0 os numeros de {a1, · · · , an0} serao cancelados na diferenca

sn − s′n pois

s′n =

n0∑s=1

as +n∑

ks>n0

aks

sn =

n0∑s=1

as +n∑

k=n0+1

ak

logo tomando a diferenca

|sn − s′n| = |n∑

k=n0+1

ak +n∑

ks>n0

aks| ≤n∑

ks>n0

|aks|+n∑

k=n0+1

|ak| <ε

2+ε

2= ε.

Logo sn e s′n convergem para o mesmo limite.

⋆ Teorema 3 (Riemann). Se uma serie∑

an e condicionalmente convergente (nao

converge absolutamente) entao para qualquer c real, existe f : N → N bijecao tal que∑af(n) = c, isto e, se uma serie e condicionalmente convergente entao existe uma reor-

denacao dos termos de∑

an tal que o resultado da serie resulte em c.

ê Demonstracao. Como∑

pn = ∞ entao podemos somar uma quantidade sufi-

ciente de termos positivos da serie tal que a soma resulte em s1 tal que c < s1, da mesma

maneira como∑

qn = ∞ podemos somar uma quantidade suficiente de termos negativos

tais que a soma total resulte em s2 tal que s2 < c < s1. Como lim an = 0 conforme n

cresce os termos ficam cada vez menores , por isso podemos somar novamente uma quan-

tidade finita de termos positivos tais que a soma total resulte em s3 com s2 < c < s3 < s1

, seguindo esse processo criamos uma reordenacao da soma dos termos de∑

an tais que∑af(n) = c

Page 51: Series

CAPITULO 1. SERIES 50

b Propriedade 49. Se uma serie e condicionalmente convergente entao existem al-

teracoes na ordem da soma dos seus termos de modo a tornar a serie +∞ ou −∞.

êDemonstracao. Como vale∑

qn = ∞ podemos somar uma quantidade suficiente

de termos negativos da serie tal que a soma resulte em −s1 e qn seja arbitrariamente

pequeno, daı como∑

pn = ∞ somamos um numero suficiente de termos positivos para

que o resultado seja s2︸︷︷︸>0

+ A︸︷︷︸>0

> 0, como qn e pequeno somamos um numero suficiente tal

que o resultado seja s3 tal que A < s3 < s2 +A, novamente somamos uma quantidade de

termos positivos tal que o resultado seja s4 = s2+2A, somamos agora os termos negativos

tal que o resultado seja s5 com 2A < s5 < s2 + 2A, continuamos o processo, sendo que

para n suficientemente grande vale sn > p.A, onde p e natural e A > 0, logo a soma

diverge para infinito. Para que a serie seja divergente para −∞ tomamos procedimento

semelhante, porem comecando a somar termos positivos ate que pn seja pequeno e depois

comecamos a somar os termos negativos.

$ Corolario 20. Somente as series absolutamente convergentes sao comutativamente

convergentes. Se uma serie e comutativamente convergente entao ela e absolutamente

convergente e incondicionalmente convergente. Se qualquer rearranjo da serie converge

ela e absolutamente convergente e todos rearranjos convergem para mesma soma.

Z Exemplo 42. Reordene os termos da serie∞∑k=1

(−1)k

kde modo que sua soma se

torne zero.

Demonstrar que (hipotese)

−1

n< s(2n) =

n∑k=1

1

2k − 1−

4n∑k=1

1

2k< 0 < s2n−1 =

n∑k=1

1

2k − 1−

4n−4∑k=1

1

2k<

1

n

daı lim sn = 0 , sn e uma reordenacao da serie∑ (−1)k

k.

m Definicao 9 (Sequencia somavel). Uma sequencia (an) e somavel com soma s quando

� ∀ ε > 0, existe J0 ⊂ N tal que ∀ J ⊂ N finito com J0 ⊂ J tem-se |∑k∈J

ak − s| < ε.

b Propriedade 50. Se (an) e somavel entao para toda bijecao f : N → N , (bn) dada

por bn = af(n) e somavel com a mesma soma.

Page 52: Series

CAPITULO 1. SERIES 51

ê Demonstracao. Como (an) e somavel entao dado ε > 0 existe j1 ⊂ N finito tal

que ∀ A j ⊂ N com J1 ⊂ j tem-se

|∑k∈j

ak − s| < ε.

Tomamos j0 ⊂ N tal que f(j0) = j1, daı f(j0) = j1 ⊂ j. Se j0 ⊂ j entao f(j0) = j1 ⊂f(j) que implica

|∑

k∈f(j)

ak − s| = |∑k∈j

af(k) − s| = |∑k∈j

bk − s| < ε

b Propriedade 51. (an) e somavel com soma s ⇔ a serie∑

an e absolutamente

convergente e vale∑

an = s.

ê Demonstracao. Adotaremos a notacao sj =∑k∈j

ak, lembrando que j e um

conjunto finito.

⇒ Vamos mostrar que o conjunto das somas finitas e limitado e daı a serie ira convergir

absolutamente , por resultado ja demonstrado.

Dado ε = 1 existe j0 ∈ N finito tal que ∀ j com j0 ⊂ j ⇒ |s− sj| < 1. Denotaremos

a =∑k∈j0

|ak|. Seja A ⊂ N um conjunto finito arbitrario, por identidade de conjuntos vale

A ∪ j0 = (j0 \ A) ∪ A sendo que essa uniao e disjunta, daı tomando a soma sobre esses

conjuntos finitos segue∑k∈A∪j0

ak =∑

k∈j0\A

ak +∑k∈A

ak ⇒∑k∈A

ak =∑

k∈A∪j0

ak −∑

k∈j0\A

ak

sA = sA∪j0 − sj0\A

pois em geral se A e B sao conjuntos disjuntos vale que7∑

k∈A∪B

ak =∑k∈A

ak+∑k∈B

ak. Disso

segue que |s − sA| = |s − sA∪j0 + sj0\A| < |s − sA∪j0 | + |sj0\A| < 1 + a pois j0 ⊂ A ∪ j0logo |s− sA∪j0 | < 1 pela condicao de ser somavel . concluımos entao que o conjunto das

somas finitas de∑

ak e limitado, entao tal serie converge absolutamente.

⇐. Supondo agora que a serie∑

an seja absolutamente convergente com∑

an =∑pn︸ ︷︷ ︸

u

−∑

qn︸ ︷︷ ︸v

= u − v = s. Tomando uj =∑k∈J

pk, vj =∑k∈J

qk temos sj = uj − vj.

Pela convergencia absoluta de∑

an, dado ε > 0 arbitrario existe n0 ∈ N tal que, sendo

7Isso pode ser tomado como parte da definicao de soma sobre conjuntos finitos

Page 53: Series

CAPITULO 1. SERIES 52

j0 = In0 = {1, · · · , n0}, j0 ⊂ j ⇒ |u − uj| <ε

2, |v − vj| <

ε

2pela definicao de limite

aplicada as somas, daı j0 ⊂ j ⇒

|s− sj| = |uj − vj − (u− v)| ≤ |u− uj|+ |v − vj| <ε

2+ε

2= ε.

daı a sequencia e somavel.

Z Exemplo 43. Dar o exemplo de uma sequencia (xn) tal que lim∆xn = 0 e xn seja

divergente porem limitada. Tomamos x1 = 0, x2 = 1, temos um passo h = 1, tomamos

agora o passo h =−1

2, x3 =

1

2, x4 = 0, tomamos agora o passo h =

1

4e somamos ate

chegar em 1 novamente, continuamos o processo dividindo sempre o passo por 2 e fazendo

a sequencia alternar entre 0 e 1. A sequencia construıda dessa forma e divergente, pois

possui subsequencias convergindo para valores distintos, e limitada pois esta sempre em

[0, 1] e a sequencia das diferencas tende a zero |xn+1 − xn|.

1.5 Soma sobre um conjunto infinito arbitrario

m Definicao 10. Sejam A ⊂ R , f : A → R, tal que f(x) ≥ 0 para todo x ∈ A e o

conjunto

S = {∑k∈ F

f(k) |F ⊂ A | F e finito}.

se S e limitado superiormente definimos

∑k∈A

f(k) = supS

se nao

∑k∈A

f(k) = ∞

nesse caso dizemos que a serie diverge.

Page 54: Series

CAPITULO 1. SERIES 53

1.6 Series em espacos vetoriais normados

Seja E um espaco vetorial normado.

m Definicao 11 (Serie em espaco vetorial normado). Seja (xn) em E, definimos a serie∞∑k=1

xk como

∞∑k=1

xk := limn∑

k=1

xk

quando tal limite existe, dizemos que a serie e convergente , caso contrario dizemos que e

divergente.

b Propriedade 52. Se∞∑k=1

xk converge, entao limxn = 0.

1.7 Soma de Cesaro

m Definicao 12 (Media de Cesaro). Dada uma sequencia (xn) definimos a media de

Cesaro de (xn) como a sequencia (yn) dada por

yn =1

n

n∑k=1

xk

yn e a media aritmetica dos n primeiros elementos de (xn)

A seguir provaremos resultados dos quais a seguinte propriedade segue como corolario

Se lim xn = a entao lim yn = a, isto e, a operacao de tomar a media de Cesaro preserva

sequencias convergentes e seus limites.

m Definicao 13 (Cesaro somavel). Se lim

n∑k=1

xk

n= L entao a sequencia (xn) e dita

Cesaro somavel e associamos a essa sequencia o valor L como soma de Cesaro . Dizemos

que (xn) e (C, 1) somavel para L, nesse caso escrevemos

limxn = L (C, 1).

limxn = L (C, 1) ⇔ lim

n∑k=1

xk

n= L.

Page 55: Series

CAPITULO 1. SERIES 54

Toda sequencia convergente e Cesaro somavel, porem existem sequencias nao conver-

gentes que sao Cesaro somavel .

⋆ Teorema 4 (Teorema de Stolz-Cesaro). Dada uma sequencia (xn) e uma sequencia

(yn) crescente com

lim yn = ∞

e lim∆xn∆yn

= a entao limxnyn

= a.

Essa propriedade e o analogo do teorema de L’Hospital para sequencias. Lembrando

que estamos denotando ∆ como o operador que faz ∆xn = xn+1 − xn, toma a diferenca

de tais numeros consecutivos na sequencia.

ê Demonstracao.

Como lim∆xn∆yn

= a entao para todoε

3> 0 existe n0 ∈ N tal que para k > n0 tem-se

|∆xk∆yk

− a| < ε

3,

e yn > 0 (pois tende ao infinito), como (yn) e crescente vale ∆yk > 0, logo podemos

multiplicar por ele em ambos lados da desigualdade sem alterar

|∆xk − a∆yk| <ε

3∆yk,

aplicando a soman−1∑

k=n0+1

em ambos lados e usando desigualdade triangular do tipo |∑

xk| ≤∑|xk|, segue que

|n−1∑

k=n0+1

∆xk − a

n−1∑k=n0+1

∆yk| <ε

3

n−1∑k=n0+1

∆yk,

usando a soma telescopica tem-se

|xn − xn0+1 − ayn + ayn0+1)| <ε

3(yn − yn0+1),

agora como yn > 0 dividimos por esse termo de ambos lados

|xnyn

− xn0+1

yn− a+ a

yn0+1

yn)| < ε

3(1− yn0+1

yn),

Page 56: Series

CAPITULO 1. SERIES 55

somando agora |xn0+1

yn|+ | − a

yn0+1

yn)| e usando a desigualdade triangular , temos

|xnyn

− a| < ε

3(1− yn0+1

yn)︸ ︷︷ ︸

≤1

+|xn0+1

yn|+ |ayn0+1

yn)|

tem-se que 1 − yn0+1

yn≤ 1 pois equivale a 0 ≤ yn0+1

yn, que vale pois yn0+1 e yn sao

positivos, como yn → ∞, podemos tomar para n suficientemente grande que |xn0+1

yn| < ε

3

e tambem |ayn0+1

yn)| < ε

3, usando tais desigualdades, tem-se finalmente que

|xnyn

− a| ≤ ε

3+ε

3+ε

3= ε

portantoxnyn

→ a.

b Propriedade 53. Se limzn = a e (wn) e uma sequencia de numeros positivos com

limn∑

k=1

wk = ∞ entao lim

n∑k=1

wkzk

n∑k=1

wk

= a.

ê Demonstracao. Tomamos xn =n∑

k=1

wk.zk e yn =n∑

k=1

wk entao ∆xn = wn+1.zn+1

, ∆yn = wn+1 > 0 entao yn e crescente e lim yn = ∞, temos tambem que∆xn∆yn

=

wn+1zn+1

wn+1

= zn+1 cujo limite existe e vale a entao nessas condicoes vale

limxnyn

= lim

n∑k=1

wk.zk

n∑k=1

wk

= a.

$ Corolario 21. Tomando wn = 1 entaon∑

k=1

wk = n e seu limite e infinito, tomando

uma sequencia (zn) tal que lim zn = a entao segue que

lim

n∑k=1

zk

n= a

, isto e, se lim zn = a entao lim

n∑k=1

zk

n= a.

Provamos entao que se limxn = a entao lim xn = a (C, 1).

Page 57: Series

CAPITULO 1. SERIES 56

Z Exemplo 44. Tomando zn =1

ntem-se lim zn = 0 e daı

lim

n∑k=1

1k

n= 0 = lim

Hn

n.

Z Exemplo 45. Tomando zn = a1n com a > 0 tem-se lim zn = 1 e daı

lim

n∑k=1

a1k

n= 1.

b Propriedade 54 (Stolz-Cesaro para limite infinito). Seja (bn) crescente e ilimitada .

Se lim∆an∆bn

= ∞ entao limanbn

= ∞

ê Demonstracao. Para qualquer A > 0 existe n0 ∈ N tal que k > n0 implica

∆ak∆bk

> A,

como ∆bk > 0 e bk > 0, logo tem-se

∆ak > A∆bk,

aplicandon∑

k=n0+1

segue por soma telescopica

an+1 − an0+1 > A.(bn+1 − bn0+1)

an+1 > an0+1 + A.(bn+1 − bn0+1)

an+1

bn+1

>an0+1

bn+1

+ A.(1− bn0+1

bn+1

) > A

para n grande, daı

limanbn

= ∞.

Z Exemplo 46. A reciproca da propriedade nem sempre vale, yn = n, xn = (−1)n

vale limxnyn

= lim(−1)n

n= 0 e lim

∆xn∆yn

= lim(−2)(−1)n

1nao existe.

b Propriedade 55. Se lim an = ∞ e an > 0∀ n ∈ N entao limn∑

k=1

akn

= ∞.

Page 58: Series

CAPITULO 1. SERIES 57

ê Demonstracao. Essa prova vale mesmo se (an) nao tem a restricao de an > 0

. Aplicamos o teorema de Stolz-Cesaro para limite infinito . an =n∑

k=1

ak , bn = n e

crescente e ilimitada e vale ∆n∑

k=1

ak = an+1 , ∆n = 1 logo

lim∆an∆n

= lim an+1 = ∞

entao limn∑

k=1

akn

= ∞.

$ Corolario 22. Esse resultado diz que se lim xn = ∞ entao lim xn = ∞ ∈ (C, 1)

ê Demonstracao.[2]

∀ A > 0 ∃ n0 ∈ N tal que para n > n0 tem-se an > 2A entao para n > 2n0 ( que

implican− n0

n>

1

2) vale

n∑k=1

ak

n≥

n∑k=n0+1

2A

n= 2A

n− n0

n≥ 2A

2= A

logo

lim

n∑k=1

ak

n= ∞.

$ Corolario 23. Se lim xn = ∞ e nao vale xn > 0 ∀ n ∈ N entao a propriedade

tambem vale pois existe n0 ∈ N tal que para n > n0 tem-se xn > 0 , daı

n∑k=1

ak

n=

n0∑k=1

ak

n+

n∑k=n0+1

ak

n=

n0∑k=1

ak

n+

n−n0∑k=1

xk︷ ︸︸ ︷ak+n0

n

assim se define uma nova sequencia (xn) que satisfaz as propriedades do resultado anterior

.

b Propriedade 56.

lim ln(n+ 1)− ln(n) = 0.

ê Demonstracao.

lim ln(n+ 1

n) = lim ln(1 +

1

n) = ln(1) = 0.

Page 59: Series

CAPITULO 1. SERIES 58

b Propriedade 57.

limln(n+ 1)

n= 0.

ê Demonstracao. Tomando yn = n e xn = ln(n + 1) vale que ∆yn = 1 > 0 e

lim yn = ∞, ∆xn = ln(n+ 1

n) logo

lim∆yn∆xn

= lim ln(n+ 1

n) = 0

logo limln(n+ 1)

n= 0.

Z Exemplo 47. Calcule o limite

limn∑

k=1

k ln(k)

n2 ln(n).

Tomando xn =n∑

k=1

k ln(k) e yn = n2 ln(n) vale lim yn = m∞ e ∆yn > 0, logo por

Stolz-Cesaro podemos avaliar o limite

lim(n+ 1) ln(n+ 1)

(n+ 1)2 ln(n+ 1)− n ln(n)

como para n grande ln(n+ 1) ≈ ln(n)

lim(n+ 1) ln(n+ 1)

(n+ 1)2 ln(n+ 1)− n ln(n)= lim

(n+ 1)

(n+ 1)2 − n= lim

(n+ 1)

2n+ 1=

1

2.

Logo

limn∑

k=1

k ln(k)

n2 ln(n)= 1.

b Propriedade 58. Se (xn) e limitada entao (xn) (C, 1) tambem e limitada, e no mesmo

intervalo .

ê Demonstracao. Existem c1, c2 tais que c2 < xk < c1, daı somamos em ambos

lados nc2 <n∑

k=1

xk < nc1 dividindo por n segue

c2 <

n∑k=1

xk

n< c1.

Page 60: Series

CAPITULO 1. SERIES 59

1.7.1 Serie de Grandi

m Definicao 14 (Serie de Grandi). A serie de Grandi e a serie∞∑k=0

(−1)k.

Luigi Guido Grandi (1671 − 1742) foi um padre italiano , filosofo, matematico, e

engenheiro.

$ Corolario 24. A serie de Grandi e divergente, pois nao existe lim(−1)n.

b Propriedade 59. A serie de Grandi e Cesaro somavel e possui soma de Cesaro de

valor1

2.

ê Demonstracao.n∑

k=0

(−1)k =(−1)n

2+

1

2

daın∑

k=1

(−1)k

2+

1

2=n

2+

(−1)n + 1

2

daı limn

2n+

(−1)n + 1

2n=

1

2.

b Propriedade 60. Suponha que vale L−ε <n∑

k=1

xkn

para k ≤ n, se vale L+ε(m+ n

m− n) <

xk para k > n entao

L+ ε <m∑k=1

xkm.

ê Demonstracao. Da desigualdade L+ε(m+ n

m− n) < xk , aplicando

m∑k=n+1

em ambos

lados tem-se

(m− n)L+ ε(m+ n) <m∑

k=n+1

xk

da primeira identidade tem-se n(L− ε) <m∑k=1

xk somando as desigualdades segue

(m)(L+ ε) <m∑k=1

xk

daı

L+ ε <m∑k=1

xkm.

Page 61: Series

CAPITULO 1. SERIES 60

1.8 Sequencias (C,P ) somaveis

m Definicao 15 (Metodo regular de somabilidade). Um metodo de somabilidade M e

regular se lim xn = L entao lim xn = L existe em M .

m Definicao 16 (Sequencias (C,P ) somaveis). Uma sequencia (xk) e dita (C,P )

somavel se existe L tal que

lim

n∑k=1

(n+p−1−k

n−k

)xk(

n+p−1n−1

) = L.

b Propriedade 61. Se (xk) e (C,P ) somavel entao (xk) e (C,P + 1) somavel .

ê Demonstracao.

1.9 Series de termos nao-negativos

Nesta secao iremos estudar as series de termos nao-negativos, isto e,∑

ak com ak ≥ 0.

b Propriedade 62. Sejam as series∑

ak e∑ ak

1 + ak.∑

ak converge ⇔∑ ak

1 + akconverge.

ê Demonstracao. ⇒.∑

ak converge e vale

0 ≤ ak ⇒ 1 ≤ 1 + ak ⇒ 1

1 + ak≤ 1 ⇒ ak

1 + ak≤ ak

pelo criterio de comparacao segue que∑ ak

1 + akconverge.

⇐.∑ ak

1 + akconverge entao

limak

1 + ak= 0 ⇒ lim 1− 1

ak + 1= 0 ⇒ lim

1

ak + 1= 1

daı por propriedade de limite lim ak + 1 = 1 ⇒ lim ak = 0 entao existe n0 tal que para

k > n0 tem-se ak ≤ 1

ak + 1 ≤ 2 ⇒ 1

2≤ 1

ak + 1⇒ ak

2≤ akak + 1

logo por comparacao∑

ak converge .

Page 62: Series

CAPITULO 1. SERIES 61

1.9.1 Criterio de comparacao por limite para series de termos

positivos

b Propriedade 63. 1. Sejam duas series∑

ak e∑

bk de termos positivos, se existe

limakbk

= a = 0 entao∑

ak converge ⇔∑

bk converge .

2. Se limakbk

= 0 entao a convergencia de∑

bk implica convergencia de∑

ak.

ê Demonstracao.

1. Existe n0 ∈ N tal que para k > n0 tem-se

0 < t1 < a− ε <akbk< a+ ε < t2

como bk > 0 tem-se

t1bk < ak < t2bk

aplicamos a soman∑

k=n0+1

, daı

t1

n∑k=n0+1

bk <

n∑k=n0+1

ak < t2

n∑k=n0+1

bk

usando essa desigualdade temos por comparacao que se∑

bk converge entao∑

ak

converge e se∑

ak converge entao∑

bk converge.

2. De maneira similar ao item anterior.

Existe n0 ∈ N tal que para k > n0 tem-se

0 ≤ akbk< ε < t2

como bk > 0 tem-se

0 ≤ ak < t2bk

aplicamos a soman∑

k=n0+1

, daı

0 ≤n∑

k=n0+1

ak < t2

n∑k=n0+1

bk

usando essa desigualdade temos por comparacao que se∑

bk converge entao∑

ak

converge.

Page 63: Series

CAPITULO 1. SERIES 62

Z Exemplo 48. A serie∑

sen(1

k) diverge pois

∑ 1

kdiverge e

limk→∞

sen( 1k)

1k

= 1,

pois isso equivale tomando1

k= x que x→ 0 entao caı no limite fundamental

limx→0

sen(x)

x= 1.

Notamos que sen(1

k) e positivo pois a funcao e positiva no intervalo (0,

π

2). Por isso

podemos aplicar o criterio .

b Propriedade 64. Seja (ak) uma sequencia positiva.n∑

k=1

ak converge ⇔n∑

k=1

akk∑

j=1

aj

converge.

ê Demonstracao.

� Suponha quen∑

k=1

ak converge, vamos mostrar quen∑

k=1

akk∑

j=1

aj

tambem converge.

Temos que a1 ≤k∑

j=1

aj, logo

1k∑

j=1

aj

≤ 1

a1⇒ ak

k∑j=1

aj

≤ aka1,

somando, segue que

n∑k=1

akk∑

j=1

aj

n∑k=1

ak

a1,

portanto a convergencia den∑

k=1

ak implica a convergencia den∑

k=1

akk∑

j=1

aj

e a di-

vergencia den∑

k=1

akk∑

j=1

aj

implica divergencia den∑

k=1

ak.

Page 64: Series

CAPITULO 1. SERIES 63

� Agora vamos provar que a divergencia den∑

k=1

ak, implica a divergencia den∑

k=1

akk∑

j=1

aj

. Vamos denotar Sk =k∑

j=1

aj. Temos que

m∑k=n+1

akSk

≥m∑

k=n+1

akSm

=1

Sm

(m∑k=1

ak −n∑

k=1

ak

)=

=1

Sm

(Sm − Sn) = 1− Sn

Sm

,

como Sm → ∞ podemos concluir que

∞∑k=n+1

akSk

≥ 1,

para qualquer n, logo a sequencia nao e de Cauchy e portanto nao converge. Por fim

nao podemos ter convergencia den∑

k=1

akk∑

j=1

aj

= Vn com divergencia den∑

k=1

ak = Sn,

pois a divergencia de Sn implica divergencia de Vn.

b Propriedade 65. Valem as desigualdades

n∑k=1

akk∑

t=1

at

≤ 1

a1

n∑k=1

ak,

n∑k=1

akk∑

j=1

j∑t=1

at

≤ 1

a1

n∑k=1

ak.

ê Demonstracao. A primeira ja provamos, na propriedade anterior, vamos provar

a segunda. Vamos denotar Tk =k∑

j=1

j∑t=1

at. Vale que a1 ≤ Tk, o que implica1

Tk≤ 1

a1,

multiplicando por ak e somando, segue

n∑k=1

akTk

≤ 1

a1

n∑k=1

ak.

Disso segue, que sen∑

k=1

ak converge, entaon∑

k=1

akTk

tambem converge, sen∑

k=1

akTk

diverge,

entaon∑

k=1

ak tambem diverge.

Page 65: Series

CAPITULO 1. SERIES 64

b Propriedade 66. Sejam (ak) sequencia de termos positivos e Sn =n∑

k=1

ak, entao

∞∑n=1

2an(Sn)2

converge.

ê Demonstracao. Sn e uma sequencia crescente, daı ela converge para um numero

positivo ou tende a infinito, em qualquer dos casos o limite de1

Sn

existe. Temos que

Sn−1 < Sn, daı Sn−1Sn < (Sn)2 e

1

(Sn)2≤ 1

Sn−1Sn

, multiplicando por an segue

an(Sn)2

≤ anSn−1Sn

=Sn − Sn−1

Sn−1Sn

=1

Sn−1

− 1

Sn

= −∆1

Sn−1

,

aplicando a soma neste ultimo termo, tem-se por soma telescopica

∞∑n=1

anSn−1Sn

= −∞∑n=1

∆1

Sn−1

=1

S0

− lim1

Sn−1

,

mas, como notamos, lim1

Sn−1

existe, por isso a serie∞∑n=1

anSn−1Sn

converge e daı tambem

converge∞∑n=1

an(Sn)2

por criterio de comparacao.

b Propriedade 67. Seja (ak) sequencia em (0, 1). Entao

n∑k=1

akk∑

j=1

j∑t=1

at

converge.

ê Demonstracao. Sejam Tn =n∑

j=1

j∑t=1

at , Sn =n∑

k=1

ak. Primeiro, vamos mostrar

que

(Sn)2

2≤ Tn, isto e,

(n∑

k=1

ak

)2

2≤

n∑j=1

j∑t=1

at.

Por inducao sobre n. Para n = 1, temos

a212

≤ a1 ⇔ a1 ≤ 2,

logo vale, pois a1 < 1. Suponha validade para n, vamos provar para n + 1. Usando

hipotese da inducao e que (Sn+1)2 = (Sn + an+1)

2 = (Sn)2 +2an+1Sn + (an+1)

2, segue que

(Sn+1)2

2=

(Sn)2

2+ an+1Sn +

(an+1)2

2≤ Tn + Sn + an+1 =

Page 66: Series

CAPITULO 1. SERIES 65

= Tn + Sn+1 =n∑

j=1

j∑t=1

at +n+1∑k=1

ak =n+1∑j=1

j∑t=1

at = Tn+1,

como querıamos mostrar.

Agora, de(Sn)

2

2≤ Tn, segue que

1

Tn≤ 2

(Sn)2, multiplicando por an e somando de

ambos lados, temos que∞∑n=1

anTn

≤∞∑n=1

2an(Sn)2

,

essa ultima serie converge pela propriedade anterior, logo por comparacao∞∑n=1

anTn

con-

verge.

Z Exemplo 49. Pode valer que∑

ak converge, valendo limakbk

= 0 e∑

bk nao

converge, tome por exemplo ak =1

k2, bk =

1

k,∑

bk nao converge, limakbk

= limk

k2=

lim1

k= 0 e

∑ak converge, logo a recıproca do item 2 da propriedade anterior nao vale.

Z Exemplo 50. Se∑

ak de termos positivos converge entao∑

sen(ak) tambem

converge, pois da primeira convergencia temos lim ak = 0 daı para k grande vale que

sen(ak) > 0 e vale limsen(ak)

ak= 1 entao

∑sen(ak) converge.

Podemos ainda resolver sem esse criterio, pois se 0 < |x| < π

2tem-se sen(x) < x, daı

com

0 ≥ sen(ak) < ak

e por comparacao a primeira converge.

b Propriedade 68. Seja (an) uma sequencia nao-crescente de numeros reais positivos.

Se∑

ak converge entao limnan = 0.

ê Demonstracao. Usaremos o criterio de Cauchy . Existe n0 ∈ N tal que para

n+ 1 > n0 vale

2na2n2

= na2n ≤2n∑

k=n+1

ak < ε

logo lim 2na2n = 0. Agora mostramos que a subsequencia dos ımpares tambem tende a

zero. Vale a2n+1 ≤ a2n daı 0 < (2n + 1)a2n+1 ≤ 2na2n + a2n por teorema do sanduıche

segue o resultado. Como as subsequencias pares e ımpares de (nan) tendem a zero, entao

a sequencia tende a zero.

Page 67: Series

CAPITULO 1. SERIES 66

$ Corolario 25. A serie harmonica∑ 1

kdiverge, pois (

1

n) e decrescente e vale lim

n

n=

1 = 0.

b Propriedade 69. Seja (xk) uma sequencia de numeros nao negativos com a serie∑xk convergente entao

∑x2k e convergente.

ê Demonstracao.[1] Como∑

ak e convergente, vale lim ak = 0 e daı para k > n0

vale xk < 1 que implica x2k ≤ xk logo por comparacao∑

x2k converge.

ê Demonstracao.[2] Como temos xk ≥ 0 segue tambem x2k ≥ 0, sendo entao

s(n) =n∑

k=b

x2k temos ∆s(n) = x2n+1 ≥ 0, logo s(n) e nao decrescente, se mostrarmos que

a serie e limitada superiormente teremos uma sequencia que e limitada e monotona logo

convergente. Temos que s(n) e limitada superiormente da seguinte maneira

n∑k=b

x2k ≤ (n∑

k=b

xk)(n∑

k=b

xk)

logo a serie e convergente.

$ Corolario 26. Se∑

ak e absolutamente convergente entao∑

a2k converge, usamos o

resultado anterior com xk = |ak|, entao a convergencia de∑

|ak| implica a convergencia

de∑

|ak|2 =∑

a2k.

Z Exemplo 51. Se nao vale xk > 0 entao podemos ter∑

xk convergente e∑

x2k

divergente, pois∑ (−1)k√

kconverge e

∑ 1

kdiverge.

b Propriedade 70. Se∑

ak, ak > 0 converge entao a serie∑ √

akk

tambem converge

.

ê Demonstracao. Usando a desigualdade de Cauchy

(n∑

k=1

xkyk)2 ≤ (

n∑k=1

x2k)(n∑

k=1

y2k)

com yk =1

ke xk =

√ak tem-se

(n∑

k=1

√akk

)2 ≤ (n∑

k=1

ak)(n∑

k=1

1

k2)

Page 68: Series

CAPITULO 1. SERIES 67

logon∑

k=1

√akk

√√√√(n∑

k=1

ak)(n∑

k=1

1

k2)

a serie e limitada superiormente, sendo crescente, ela converge .

$ Corolario 27. Se∑

x2k, converge entao a serie∑ xk

ktambem converge, basta usar

o resultado anterior com ak = x2k.

b Propriedade 71. Se∑

x2n e∑

y2n convergem entao∑

xn.yn converge absoluta-

mente.

ê Demonstracao. Usando a desigualdade de Cauchy

(n∑

k=1

|xk||yk|)2 ≤ (n∑

k=1

|xk|2)(n∑

k=1

|yk|2) = (n∑

k=1

x2k)(n∑

k=1

y2k)

logo por criterio de comparacao segue que∑

xn.yn converge absolutamente.

1.10 Representacao decimal

m Definicao 17 (Representacao numa base b). Dado um numero natural b > 1, a

representacao de um numero real x na forma

x =n∑

k=−∞

bkak

onde ak ∈ {0, · · · , b− 1}, e chamada de representacao na base b do numero x . Cada ak

e chamado de algarismo e k de seu ındice.

Caso x =n∑

k=−∞

bkak denotaremos tambem

x = (an · · · a0, a−1 · · · a−t · · · )b

que vamos denotar em notacao compacta

x = (ak)n(k=−∞, b).

Page 69: Series

CAPITULO 1. SERIES 68

Caso de um numero natural

x = (ak)n(b) = (a0, · · · , an)b

o ındice b para simbolizar a base, o expoente n para simbolizar que k varia de 0 ate n.

b Propriedade 72. Todo numero m ∈ N pode ser representado numa base a.

ê Demonstracao.

Pelo teorema de divisao euclidiana, se tomarmos numeros f(0) = m e a = 0 naturais,

teremos numeros f(1) e R(0) determinados univocamente, tais que f(0) = af(1) + R(0)

com 0 ≤ R(0) < a. Onde f(1) e o quociente, R(0) e o resto da divisao de f(0) por a.

Podemos assim definir uma sequencia

f(n) = af(n+ 1) +R(n)

onde R(n) e sempre o resto da divisao de f(n) por a, logo R(n) ∈ {0, · · · , a−1}, f(n+1)

e o quociente. esse tipo de recorrencia podemos encontrar a formula geral .

Vamos resolver entao essa recorrencia. Tomamos f(n) = h(n)1

an, substituindo temos

f(n)

a− R(n)

a= f(n+ 1)

h(n)

an+1− R(n)

a=h(n+ 1)

an+1

−R(n)a

an+1 = −anR(n) = ∆h(n)

aplicando o somatorio em ambos termos, variando de k = 0 ate n− 1 temos

n−1∑k=0

∆h(k) = h(n)− h(0) = −n−1∑k=0

akR(k)

h(n) = h(0)−n−1∑k=0

akR(k)

logo temos

f(n)an = h(0)−n−1∑k=0

akR(k)

Page 70: Series

CAPITULO 1. SERIES 69

tomando n = 0 temos

f(0) = h(0)

logo

f(n)an +n−1∑k=0

akR(k) = f(0)

se f(n) = R(n), podemos juntar ao limite superior do somatorio, ficando com

f(0) =n∑

k=0

akR(k)

Este resultado permite ver o metodo para expressar um numero em termo de potencias

de a que e chamado de base.

m Definicao 18. Um algarismos a esquerda de um algarismo at dado de

m∑k=−∞

bk

sao os algarismos ak com k > t, caso existam . Algarismos a direita de at sao os algarismos,

ak com k < t, caso existam . Dados dois algarismos at e aw distintos, w > t, os algarismos

entre esses dois sao os algarismos ak com t < k < w, caso w = t + 1 entao nao existe

algarismo entre at e aw.

m Definicao 19 (Representacao decimal de numero natural). Um numero natural pode

ser representado da forman∑

k=0

ak10k.

m Definicao 20 (Representacao decimal de um numero real). Seja dada uma sequencia

(ak)∞0 = (a0, a1, a2, · · · ) onde a0 e um inteiro qualquer e ak com k > 0 pertence ao conjunto

{0, 1, 2, 3, 4, 5, 6, 7, 8, 9}. Um numero real na forma decimal e representado por

a0, a1a2a3 · · ·

onde cada ak e chamado de dıgito do numero na forma decimal .

Page 71: Series

CAPITULO 1. SERIES 70

Para dar sentido a a0, a1a2a3 · · · como numero real, definimos

a0, a1a2a3 · · · =∞∑k=0

ak10k

= a0 +∞∑k=1

ak10k

O sistema decimal para representar numeros naturais e variante do sistema sexagesimal

utilizado pelos babilonios ha cerca de 1700 anos antes de Cristo, ele foi desenvolvido na

China e na India. Por neste sistema, todo numero ser representado por uma sequencia

formada pelos algarismos

0, 1, , 2, 3, 4, 5, 6, 7, 8, 9,

sendo em numero de 10, o sistema e portanto chamado de decimal .

O sistema decimal tambem e dito posicional, pois cada algarismo, alem de seu va-

lor intrınseco, possui um peso que lhe e atribuıdo em funcao de sua posicao dentro da

sequencia. Esse peso e uma potencia de 10 e varia como exposto acima.

Agora vamos mostrar que essa serie da representacao decimal sempre converge , logo

a0, a1a2a3 · · · representa um unico numero real.

b Propriedade 73. Cada decimal representa um unico numero real.

ê Demonstracao.∞∑k=1

ak10k

e uma serie de numeros positivos limitada superiormente

pela serie∞∑k=1

9

10kque converge para 1 entao

∞∑k=1

ak10k

converge para um numero real

pelo criterio de comparacao . O criterio de comparacao usa que uma sequencia limitada

superiormente e crescente converge para o supremo do conjunto, entao essa demonstracao

em geral necessita que o corpo em que estamos trabalhando seja completo, por exemplo,

nem toda representacao decimal converge para um numero racional.

Com isso concluımos que a0, a1a2a3 · · · =∞∑k=0

ak10k

= a0 +∞∑k=1

ak10k

= c e um numero

real .

Pela unicidade de limite o numero real que a0, a1a2a3 · · · representa e unico .

Cada a0, a1a2a3 · · · representa um e apenas um numero real.

$ Corolario 28.

0, 9999 · · · = 1

Page 72: Series

CAPITULO 1. SERIES 71

pois pela definicao de representacao decimal

0, 99 · · · = 0 +∞∑k=1

9

10k= 1

No caso mostramos que uma representacao decimal para 1 pode ser dada por a0 = 0

e ak = 9 para todo k > 0 entao associamos 0, 9999 · · · ao numero 1 .

Perceba que o numero 1 tem pelos menos duas representacoes decimais, pois 1 tambem

tem a representacao

1, 00 · · ·

pois

1, 00 · · · = 1 +∞∑k=1

0

10k= 1.

$ Corolario 29. Em geral a0, 0000 · · · = a0 e (a0 − 1), 9999 · · · = a0

pois

(a0 − 1), 9999 · · · = a0 − 1 +∞∑k=0

9

10k= a0 − 1 + 1 = a0.

Concluımos entao que todo numero inteiro a0 possui pelo menos duas representacoes

decimais

a0, 0000 · · · e (a0 − 1), 99 · · · .

Z Exemplo 52.

0, 999 · · · = 1

1, 999 · · · = 2.

m Definicao 21 (Representacoes decimais distintas). Duas representacoes decimais

a0, a1a2a3 · · · e b0, b1b2b3 · · · sao ditas distintas quando as sequencias associadas (a0, a1, a2, · · · )

e (b0, b1, b2, · · · ) sao distintas .

$ Corolario 30. Numeros reais podem ter duas representacoes decimais distintas.

Page 73: Series

CAPITULO 1. SERIES 72

Considere B o conjunto das sequencias (a0, a1, a2, · · · ) associadas a uma representacao

decimal, temos uma funcao f que associa a cada elemento de B a um numero real, definida

como

f(a0, a1, a2, · · · ) =∞∑k=0

ak10k

porem f nao e injetiva, pois existem sequencias x1 e x2 distintas tais que f(x1) = f(x2).

Podemos mostrar que f e sobrejetora, isto e, para cada x real existe uma sequencia x1 tal

que f(x1) = x.

m Definicao 22 (Dızima periodica). Uma representacao decimal a0, a1a2 · · · e dita ser

uma dızima periodica quando a sequencia dos dıgitos (ak) e periodica a partir de algum

k = n.

m Definicao 23 (Dızima periodica simples ou Dızima simples). Uma dızima periodica,

e dita ser simples, quando a sequencia dos dıgitos (ak) e periodica a partir de k = 1.

m Definicao 24 (Dızima periodica composta ou Dızima composta). Uma dızima

periodica, e dita ser composta, quando a sequencia dos dıgitos (ak) e periodica a par-

tir de k > 1.

Em R se considera a adicao usual + e o produto usual ×, que fazem de R um corpo,

alem disso se considera o limite com a norma do modulo

limxn = a⇔ ∀ ε > 0 ∃ n0 ∈ N | n > n0 ⇒ |xn − a| < ε

Se usamos outra maneira de medir distancia ao inves do modulo, nao se esta traba-

lhando em R de maneira usual, seria algo como dizer, 1 + 1 nao e 2 pois estamos usando

uma ”adicao”diferente, como por exemplo uma definida assim a +s b = (a + b).2 daı

1 +s 1 = (1 + 1)2 = 4.

Em R usando adicao, multiplicacao e norma usual, definindo a expansao decimal como

serie tem-se

0, 999 · · · = 1.

Uma colocacao comum de alguns estudantes e que 0, 999 · · · nao e 1 e sim tende a 1, o

que nao e verdade, pois 0, 999 · · · nao e uma sequencia dessa forma nao faz sentido dizer

Page 74: Series

CAPITULO 1. SERIES 73

que ele tende a 1, 0, 999 · · · e o limite de uma sequencia de numeros reais, por definicao,

sendo portanto um numero real.

b Propriedade 74. x e racional ⇔ possui representacao periodica.

ê Demonstracao.

⇒). Se x e racional x =p

q, por divisao euclidiana p = a0q + r0 logo

x = a0 +r0q.

Existe s1 mınimo tal que 10s1r0 ≥ q, daı por divisao euclidiana 10s1r0 = as1q + rs1 , entao

x = a0 +r010

s1

q10−s1 = a0 + as110

−s1 +rs1q10−s1

vale que as1 < 10 por minimalidade de s1 , pois caso contrario se as1 ≥ 10 entao

10s1r0 = as1q + rs1 ≥ 10.q

e por isso 10s1−1r0 ≥ q contradizendo a minimalidade de s1. Por isso as1 e inteiro no

conjunto {0, 1, 2, 3, 4, 5, 6, 7, 8, 9}. Repetimos o procedimento com rs1

x = a0 + as110−s1 + as210

−s2 +rs2q10−s2 ,

rsk e o resto da divisao de um numero por q, ele pode assumir os valores {0, 1, · · · , q−2, q − 1}, um numero finito de valores, entao para algum k o numero rsk deve ser igual

a algum outro rst com k > t, daı o processo para se obter ask+1e o mesmo para se obter

ast+1 e os numeros comecam a se repetir na sequencia da expansao decimal.

10sk+1 rsk︸︷︷︸=rst

≥ q ⇒ sk+1 = st+1

10sk+1 rsk︸︷︷︸=rst

= ask+1︸︷︷︸=ast+1

q + rsk+1︸︷︷︸=rst+1

por isso a representacao se torna periodica. Um numero racional possui representacao

decimal periodica.

⇐). Um numero com representacao decimal periodica representa um numero decimal

. Um numero com representacao decimal periodica e da forma

a0, a1 · · · at︸ ︷︷ ︸parte nao periodica

at+1 · · · at+pat+1 · · · · · · at+p

Page 75: Series

CAPITULO 1. SERIES 74

a0+t∑

k=1

ak10−k+at+1(10

−(t+1)+10−(t+p+1)+10−(t+2p+1)+· · · )+at+2(10−(t+2)+10−(t+p+2)+10−(t+2p+2)+· · · )+

+ · · ·+ at+p(10−(t+p) + 10−(t+p+p) + 10−(t+2p+p) + · · · )

onde cada parcela e racional, entao a soma resultante e um numero racional.

at+p(10−(t+p) + 10−(t+p+p) + 10−(t+2p+p) + · · · ) = at+p10

−(t+p)(1 + 10−(p) + 10−(2p) + · · · )

o numero∞∑k=0

10−kp e racional logo todas parcelas sao realmente racionais.

Z Exemplo 53. Achar dızima de2

11. Temos que

2

11=

20.10−1

11=

(11

11+

9

11

)10−1 = 1.10−1 +

9

1110−1 =

= 1.10−1 +90

1110−2 = 1.10−1 + (

8.11

11+

2

11)10−2 = 1.10−1 + 8.10−2 +

2

1110−2 =

como aparece novamente o termo2

11as expressoes comecam a se repetir . Entao temos

que

2

11= 0, 1818181818 · · · = 0, 18.

1.11 Teste da integral para convergencia de series

b Propriedade 75. Seja f : [1,∞) → R+ decrescente. Nessas condicoes

∞∑k=1

f(k) <∞ ⇔∫ ∞

1

f(t)dt <∞.

Se a serie converge para s, vale a estimativa

∫ ∞

n+1

f(t)dt ≤ s− sn ≤∫ ∞

n

f(t)dt

onde sn =n∑

k=1

f(k).

Page 76: Series

CAPITULO 1. SERIES 75

Em especial valem as desigualdades

s(n)− f(1) ≤∫ n

1

f(t)dt ≤ s(n− 1)

f(k) ≤∫ k

k−1

f(t)dt ≤ f(k − 1).

ê Demonstracao. De

m(b− a) ≤∫ b

a

f(t)dt ≤M(b− a)

onde M,m sao o supremo e ınfimo de f em [a, b], se tomamos o intervalo [k − 1, k] com

f decrescente essa identidade implica que

f(k) ≤∫ k

k−1

f(t)dt ≤ f(k − 1)

aplicando a soman∑

k=2

tem-se

n∑k=1

f(k)− f(1) = sn − f(1) ≤∫ n

1

f(t)dt ≤n∑

k=2

f(k − 1) =n−1∑k=1

f(k) = s(n− 1)

s(n)− f(1) ≤∫ n

1

f(t)dt ≤ s(n− 1)

portanto segue o resultado de convergencia.

Da desigualdade f(k) ≤∫ k

k−1

f(t)dt ≤ f(k − 1) aplicandom∑

n+1

resulta

s(m)−s(n) ≤∫ m

n

f(t)dt ≤ s(m−1)−s(n−1) ⇒∫ m

n

f(t)dt ≤ s(m)−s(n) ≤∫ m

n

f(t)dt

tomando m→ ∞ segue ∫ ∞

n+1

f(t)dt ≤ s− sn ≤∫ ∞

n

f(t)dt.

Z Exemplo 54. Com o teste da integral podemos novamente observar que a serie

harmonica diverge

f com f(t) =1

te decrescente e integravel, logo podemos aplicar o teste da integral∫ n

1

1

tdt = ln(n) → ∞

Page 77: Series

CAPITULO 1. SERIES 76

logo a serie harmonica diverge.

Para outras somas do tipo1

kpcom p = 1, podemos tambem aplicar o teste da integral∫ n

1

1

tpdt =

n−p+1

−p+ 1− 1−p+1

−p+ 1

que diverge se −p+ 1 > 0, p < 1 e converge se −p+ 1 < 0, 1 < p.

$ Corolario 31. Usando f(k) =1

ke a desigualdade s(n + 1) − f(1) ≤

∫ n+1

1

f(t)dt ≤

s(n), temosn+1∑k=1

1

k− 1 ≤

∫ n+1

1

1

tdt = ln(n+ 1) ≤

n∑k=1

1

k

portanto

Hn − 1 < Hn+1 − 1 ≤ ln(n+ 1) ≤ Hn

a desigualdade da direita implica 0 < Hn− ln(n+1) e a desigualdade da esquerda implica

Hn − ln(n+ 1) < 1, logo temos

0 < Hn − ln(n+ 1) < 1.

Da desigualdade

∫ n+2

n+1

f(t)dt ≤ f(n+ 1) segue que

ln(n+ 2)− ln(n+ 1) ≤ 1

n+ 1⇒ Hn − ln(n+ 1) ≤ Hn+1 − ln(n+ 2)

logo a sequencia de termo xn = Hn− ln(n+1) e monotona limitada e por isso convergente.

m Definicao 25 (Constante de Euler-Mascheroni). O limite limHn − ln(n + 1) = γ e

chamada de constante de Euler-Mascheroni, e um problema em aberto saber se tal numero

e racional ou irracional.

1.11.1 Sequencia de variacao limitada

m Definicao 26 (Sequencia de variacao limitada). Uma sequencia (xn) tem variacao

limitada quando a sequencia (vn) com

vn =n∑

k=1

|∆xk| e limitada.

Page 78: Series

CAPITULO 1. SERIES 77

b Propriedade 76. Se (xn) tem variacao limitada entao (vn) converge.

ê Demonstracao. (vn) e limitada e nao-decrescente, pois ∆vn = |∆xn+1| ≥ 0, logo

e convergente.

b Propriedade 77. Se (xn) tem variacao limitada entao existe limxn.

ê Demonstracao. A serie∞∑k=1

|∆xk| converge portanto∞∑k=1

∆xk converge absoluta-

mente e vale

xn − x1 =n−1∑k=1

∆xk ⇒ xn =n−1∑k=1

∆xk + x1

logo xn e convergente.

Z Exemplo 55. Se |∆xn+1| ≤ c|∆xn| ∀ n ∈ N com 0 ≤ c < 1 entao (xn) possui

variacao limitada. Definimos g(k) = |∆xk| logo a desigualdade pode ser escrita como

g(k + 1) ≤ cg(k), Qg(k) ≤ c aplicamosn−1∏k=1

de ambos lados, daı

g(n) = |∆xn| ≤ cn−1g(1)

somando em ambos lados temos

n∑k=1

|∆xk| ≤n∑

k=1

ck−1g(1)

como o segundo termo converge por ser serie geometrica segue que (xn) e de variacao

limitada, logo converge.

b Propriedade 78. (xn) tem variacao limitada ⇔ xn = yn − zn onde (yn) e (zn) sao

sequencias nao-decrescentes limitadas.

ê Demonstracao.

⇐).

Seja xn = yn − zn onde (yn) e (zn) sao sequencias nao-decrescentes limitadas, entao

xn tem variacao limitada.

vn =n∑

k=1

|∆xk| =n∑

k=1

|∆yk −∆zk| ≤n∑

k=1

|∆yk︸︷︷︸≥0

|+n∑

k=1

|∆zk︸︷︷︸≥0

| =n∑

k=1

∆yk +n∑

k=1

∆zk =

Page 79: Series

CAPITULO 1. SERIES 78

= (yn+1 − y1) + (zn+1 − z1) < M

pois (yn) e (zn) sao limitadas, logo (vn) e limitada, isto e, (xn) tem variacao limitada.

⇒). Dada (xn) com variacao limitada. (xn) tem variacao limitada ⇔ (xn + c) tem

variacao limitada, pois ∆ aplicado as duas sequencias tem o mesmo valor. Escrevemos

xn − x1 =n−1∑k=1

∆xk

Para cada n definimos Pn o conjunto dos k da soman−1∑k=1

∆xk tais que ∆xk ≥ 0 e Nn

o conjunto dos k da mesma soma tais que ∆xk < 0, com isso temos uma particao do

conjunto dos ındices e vale

xn − x1 =n−1∑k=1

∆xk =∑k∈Pn

∆xk︸ ︷︷ ︸yn

−∑k∈Nn

(−∆xk)︸ ︷︷ ︸zn

(yn) e nao decrescente, pois yn+1 = yn caso nao seja adicionado ındice a Pn+1 em relacao

a Pn e yn+1 ≥ yn caso seja adicionado um ındice a Pn+1, pois adicionamos um termo da

forma ∆xk ≥ 0 o mesmo para (zn).

(yn) e limitada pois

∑k∈Pn

∆xk ≤n−1∑k=1

|∆xk| =∑k∈Pn

|∆xk|+∑k∈Nn

|∆xk| =∑k∈Pn

∆xk +∑k∈Nn

(−∆xk) < M

da mesma maneira (zn) e limitada.

Z Exemplo 56. Existem sequencias convergentes que nao possuem variacao limitada,

como por exemplo xn =n−1∑k=1

(−1)k

k, que e convergente porem ∆xn =

(−1)n

n⇒ |∆xn| =

1

n

en−1∑k=1

1

knao e limitada.

Z Exemplo 57. Seja (xn) definida como x1 = 1, xn+1 = 1 +1

xn, entao vale que

|∆xn+1| ≤1

2|∆xn|.

� Primeiro vale que xn ≥ 1 para todo n pois vale para n = 1, supondo validade para

n, entao vale para n+ 1, pois xn+1 = 1 +1

xn.

Page 80: Series

CAPITULO 1. SERIES 79

� Vale que |xn+1xn| ≥ 2 para todo n, pois, substituindo xn+1 = 1 +1

xnisso implica

que xn+1xn ≥ xn + 1 ≥ 2.

� De |xn+1xn| ≥ 2 segue que | 1

xn+1xn| ≤ 1

2, multiplicando por |xn+1 − xn| em ambos

lados segue que

|xn − xn+1

xn+1xn| ≤ |xn+1 − xn|

2

| 1

xn+1

− 1

xn| = | (1 + 1

xn+1

)︸ ︷︷ ︸xn+2

− (1 +1

xn)︸ ︷︷ ︸

xn+1

| ≤ |xn+1 − xn|2

portanto |∆xn+1| ≤1

2|∆xn| portanto a sequencia e convergente. Calculamos seu

limite lim xn = a

a = 1 +1

a⇔ a2 − a− 1 = 0

cujas raızes sao1±

√5

2, ficamos com a raiz positiva pois a sequencia e de termos

positivos, logo

limxn =1 +

√5

2.

1.12 Series em espacos vetoriais normados

b Propriedade 79. Se S =∞∑k=1

ak converge entao lim ak = 0.

ê Demonstracao. Sn+1−Sn = an+1, tomando o limite, temos que limSn = S e daı

S − S = 0 = lim an+1.

b Propriedade 80 (Criterio de Cauchy para series). Em Rn. Uma serie S(n) =n∑

k=1

ak

converge ⇔ se para todo ε > 0 existe n0 ∈ N tal que para m > n− 1 > n0 temos

|m∑

k=n

ak| < ε.

ê Demonstracao.

Page 81: Series

CAPITULO 1. SERIES 80

Em Rn S(n) =n∑

k=1

ak converge ⇔ S(n) converge como sequencia ⇔ S(n) e de cauchy

⇔ para ∀ ε > 0 existe n0 ∈ N tal que m ≥ n− 1 > n0 tem-se

|S(m)− S(n− 1)| < ε

|m∑k=1

ak −n−∑k=1

ak| = |m∑

k=n

ak +n−1∑k=1

ak −n∑

k=1

ak| < ε

|m∑

k=n

ak| < ε.

b Propriedade 81. Em Rn. Sen∑

k=1

||ak|| converge entaon∑

k=1

ak tambem converge.

ê Demonstracao. Por desigualdade triangular vale que

||m∑

k=n

ak|| ≤m∑

k=n

||ak||

comon∑

k=1

||ak|| e de cauchy, para qualquer ε > 0 existem m ≥ n − 1 > n0 tais que

m∑k=n

||ak|| < ε logo tambem vale ||m∑

k=n

ak|| < ε portanto a serien∑

k=1

ak e de Cauchy e daı

convergente.

b Propriedade 82 (Criterio de comparacao). Se existe (ck) em R ,n∑

k=1

ck convergente

e ||ak|| < ck para k > n0 entaon∑

k=1

ak converge.

êDemonstracao. Vamos mostrar quen∑

k=1

||ak|| converge, daı pelo resultado anterior

n∑k=1

ak tambem converge.

s(n) =n∑

k=n0+1

||ak||, define uma sequencia crescente limitada superiormente por∞∑

k=n0+1

ck

logo e convergente. A sequencia e crescente pois

s(n+ 1)− s(n) = ||an+1|| ≥ 0

Page 82: Series

CAPITULO 1. SERIES 81

e limitada superiormente pois de ||ak|| < ck segue aplicando a soma em ambos lados que

n∑k=n0+1

||ak|| <n∑

k=n0+1

ck

perceba quen∑

k=1

||ak|| e uma serie de numeros reais, pois a norma de um vetor do Rn e

um numero real.

b Propriedade 83 (Criterio de Dirichlet para series em Rn.). Sen∑

k=1

ak e uma serie em

Rn com somas parciais limitadas, (bk) decrescente de numeros reais com limite nulo entao

n∑k=1

akbk

converge.

ê Demonstracao. A serien∑

k=1

akbk converge ⇔ converge coordenada a coordenada,

as coordenadas de ak ∈ Rn sao limitadas, logo podemos aplicar o criterio de Dirichlet

para series reais em cada coordenada o que implica a convergencia da serie.

1.13 Produto de series

m Definicao 27 (Produto de Cauchy). Dadas duas series∞∑k=0

ak e∞∑k=0

bk definimos seu

produto como a serie∞∑k=0

ck

onde

ck =n∑

k=0

akbnk.

Para o proximo teorema vamos demonstrar inicialmente que

b Propriedade 84 (Revertendo a ordem -Soma de elementos de uma matriz triangular

superior). Vale a propriedade

n∑k=a

k∑j=a

f(k, j) =n∑

j=a

n∑k=j

f(k, j).

Page 83: Series

CAPITULO 1. SERIES 82

ê Demonstracao. Definimos g(k, j) = 0 se j > k e g(k, j) = f(k, j) caso contrario,

daı completamos a soma

n∑k=a

k∑j=a

f(k, j) =n∑

k=a

(k∑

j=a

g(k, j) +n∑

j=k+1

g(k, j)) =n∑

k=a

n∑j=a

g(k, j) =

trocando a ordem da soma

=n∑

j=a

n∑k=a

g(k, j) =n∑

j=a

(

j−1∑k=a

g(k, j)︸ ︷︷ ︸0

+n∑

k=j

g(k, j)) =

=n∑

j=a

n∑k=j

g(k, j)︸ ︷︷ ︸f(k,j)

.

Caso especial se a = 0

n∑k=0

k∑j=0

f(k, j) =n∑

j=0

n∑k=j

f(k, j).

A identidaden∑

k=1

k∑j=1

a(k, j) =n∑

j=1

n∑k=j

a(k, j)

pode ser interpretada como a soma dos elementos de uma matriz triangular superior

a(1,1) a(2,1) a(3,1) · · · a(n,1)

0 a(2,2) a(3,2) · · · a(n,2)

0 0 a(3,3) · · · a(n,3)...

...... · · · ...

0 0 0 0 a(n,n)

na primeira soma fixamos a linha e somamos os elementos das colunas, na segunda fixamos

a coluna e somamos os elementos da linha.

No calculo de integrais temos resultado similar∫ n

a

∫ x

a

f(x, y)dydx =

∫ n

a

∫ n

y

f(x, y)dxdy.

b Propriedade 85. Vale que

n∑k=0

k∑s=0

asbk−s =n∑

k=0

akBn−k.

Page 84: Series

CAPITULO 1. SERIES 83

Onde Bn =n∑

s=0

bs, onde (bk) e (ak) sao sequencias quaisquer.

ê Demonstracao.

Revertendo a ordem da soman∑

k=0

k∑s=0

asbk−s temos

n∑k=0

k∑s=0

asbk−s =n∑

s=0

k∑k=s

asbk−s =n∑

s=0

as

n∑k=s

bk−s =n∑

s=0

as

n−s∑k=0

bk =n∑

s=0

asBn−s,

logo esta provado .

b Propriedade 86 (Teorema de Mertens). Se uma das series∞∑k=0

ak = A ou∞∑k=0

bk = B

converge absolutamente, entao∞∑k=0

ck o produto de Cauchy das series, converge para AB

.

ê Demonstracao. Tomamos An =n∑

k=0

ak, Bn =n∑

k=0

bk, Cn =n∑

k=0

ck, tn = Bn − B

onde

ck =k∑

s=0

asbk−s.

Vamos supor quen∑

k=0

ak e absolutamente convergente, logon∑

k=0

|ak| converge e portanto∑|ak| e limitada, digamos por um numero real M > 0.

Podemos escrever

Cn =n∑

k=0

ck =n∑

k=0

k∑s=0

asbk−s =n∑

k=0

akBn−k =

=n∑

k=0

ak(tn−k +B) = Bn∑

k=0

ak +n∑

k=0

aktn−k =

= BAn +n∑

k=0

aktn−k︸ ︷︷ ︸yn

.

Entao temos que mostrar que yn → 0. Sabemos que lim tn = limBn −B = 0 podemos

tomar n0 tal que n > n0 implica |tn| <ε

2Mlogo

|yn| ≤ |n0∑k=0

an−ktk|+|n∑

k=n0+1

an−ktk| ≤ |n0∑k=0

an−ktk|+n∑

k=n0+1

|an−k|︸ ︷︷ ︸≤M

|tk|︸︷︷︸≤ ε

2M

≤n0∑k=0

|an−k| |tk|+ε

2≤

Page 85: Series

CAPITULO 1. SERIES 84

como tn → 0 entao (tn) e limitada, digamos por M1 > 0, lembrando tambem que an → 0

, entao existe n1 ∈ N tal que para n−n0 > n1, isto e, n > n1 +n0, tem-se |an| ≤ε

2n0M1

,

juntando tais fatos na desigualdade anterior tem-se que

≤n0∑k=0

|an−k|︸ ︷︷ ︸≤ ε

2n0M1

|tk|︸︷︷︸M1

2≤ ε

2+ε

2= ε

logo temos lim |yn| = 0 o que prova o resultado.

b Propriedade 87 (Teorema de Abel). Se as series∞∑k=0

ak,∞∑k=0

bk,∞∑k=0

ck convergem

para A, B e C e

ck =k∑

s=0

akbn−k

entao C = AB.

ê Demonstracao.

1.14 Series e desigualdade das medias

b Propriedade 88. Sejam m sequencias (a1,k) · · · (am,k) de numeros nao negativos, que

formam series convergentes, entao a serie

∞∑k=1

m

√√√√ m∏t=1

at,k converge.

ê Demonstracao. Usamos a desigualdade entre media aritmetica e geometrica, que

garante

m

√√√√ m∏t=1

at,k ≤m∑k=1

at,km

a soma dos termos da direita converge , pois a soma finita de series convergente converge

e podemos trocar a ordem dos somatorios, entao

∞∑k=1

m

√√√√ m∏t=1

at,k ≤m∑k=1

∞∑k=1

at,k

m

logo por criterio de comparacao∞∑k=1

m

√√√√ m∏t=1

at,k converge .

Page 86: Series

CAPITULO 1. SERIES 85

1.15 Extensao do conceito de serie para−∞∑k=1

ak.

m Definicao 28 (−∞∑k=1

ak). Seja ak : Z → R.

Extendemos o conceito de soma∑

pela recorrencia

b∑k=a

ak =

p∑k=a

ak +b∑

k=p+1

ak.

temosa−1∑k=a

ak = 0 que e a soma vazia, tomando b = a− 1 segue que

a−1∑k=a

ak = 0 =

p∑k=a

ak +a−1∑

k=p+1

ak

tomando agora a = 1 e p = m segue

0 =m∑k=1

ak +0∑

k=m+1

ak ⇒m∑k=1

ak = −0∑

k=m+1

ak

tomando agora m = −n tem-se

−n∑k=1

ak = −0∑

k=−n+1

ak

com n ≥ 1 a soma −0∑

k=−n+1

ak esta bem definida e assim fica definida tambem a soma

−n∑k=1

ak.

−0∑

k=−n+1

ak = −(a1−n + · · ·+ a0) = −n−1∑k=0

a(−k)

logo−n∑k=1

ak = −n−1∑k=0

a(−k)

aplicando limn→∞

temos

−∞∑k=1

ak = −∞∑k=0

a(−k).

Page 87: Series

CAPITULO 1. SERIES 86

$ Corolario 32.−∞∑k=1

ak converge ⇔0∑

k=−∞

ak converge, pois essa segunda e∞∑k=0

a(−k) .

b Propriedade 89. Se−∞∑k=1

ak converge, entao limk→∞

a(−k) = 0.

ê Demonstracao. Como vale a igualdade−∞∑k=1

ak = −∞∑k=0

a(−k) a segunda serie e

convergente implica limk→∞

a(−k) = 0.